Appearance
❓:Let ( x_1, ldots, x_n > 0 ) with ( x_1 cdot x_2 cdot ldots cdot x_n = 1 ). Show that[ x_1^{n-1} + x_2^{n-1} + ldots + x_n^{n-1} geq frac{1}{x_1} + frac{1}{x_2} + ldots + frac{1}{x_n} ]
🧠:<think>Alright, so I've got this inequality to prove: for positive numbers ( x_1, x_2, ldots, x_n ) with their product equal to 1, I need to show that the sum of each ( x_i ) raised to the power of ( n-1 ) is at least as big as the sum of the reciprocals of each ( x_i ). That is,[x_1^{n-1} + x_2^{n-1} + ldots + x_n^{n-1} geq frac{1}{x_1} + frac{1}{x_2} + ldots + frac{1}{x_n}]Hmm, okay. Let's start by understanding the given condition: the product of all ( x_i ) is 1. That might be useful in simplifying or manipulating the inequality somehow.First, I recall that inequalities often involve techniques like AM-GM (Arithmetic Mean-Geometric Mean) inequality, Cauchy-Schwarz, or Holder's inequality. Maybe one of these could be applicable here.Let me think about AM-GM. The AM-GM inequality states that for non-negative real numbers, the arithmetic mean is greater than or equal to the geometric mean. Since all ( x_i ) are positive, AM-GM is definitely a candidate.But in this case, we have sums of different powers: ( x_i^{n-1} ) on the left and ( frac{1}{x_i} ) on the right. Maybe I can relate these terms somehow.Wait, since the product of all ( x_i ) is 1, perhaps taking reciprocals or using logarithms could help. But I'm not sure about logarithms here because they might complicate things.Let me try to manipulate the inequality directly. If I multiply both sides by the product ( x_1 x_2 ldots x_n ), which is 1, the inequality remains the same. But maybe that's not helpful.Alternatively, perhaps I can express ( frac{1}{x_i} ) in terms of the other variables. Since ( x_1 x_2 ldots x_n = 1 ), then ( frac{1}{x_i} = x_1 x_2 ldots x_{i-1} x_{i+1} ldots x_n ). That is, the product of all variables except ( x_i ).So, the right-hand side of the inequality can be written as:[sum_{i=1}^n x_1 x_2 ldots x_{i-1} x_{i+1} ldots x_n]Hmm, interesting. So now both sides of the inequality are sums of products of variables, just with different exponents.On the left, each term is ( x_i^{n-1} ), which is like multiplying ( x_i ) by itself ( n-1 ) times. On the right, each term is the product of all variables except ( x_i ), which is also ( n-1 ) variables multiplied together.So, both sides are sums of products of ( n-1 ) variables, just arranged differently. Maybe there's a way to compare these two sums.I remember that in symmetric inequalities, sometimes rearrangement or Muirhead's inequality can be used when dealing with symmetric sums. Muirhead's inequality applies to symmetric sums where one sequence majorizes another.Let me recall Muirhead's condition. If we have two sequences of exponents, say ( (a_1, a_2, ldots, a_n) ) and ( (b_1, b_2, ldots, b_n) ), and if ( a ) majorizes ( b ), then the symmetric sum for ( a ) is greater than or equal to the symmetric sum for ( b ).In our case, the left-hand side has exponents ( (n-1, 0, 0, ldots, 0) ) for each term, and the right-hand side has exponents ( (1, 1, ldots, 1, 0) ) for each term (since each term is missing one variable). Wait, actually, for the right-hand side, each term is missing one variable, so each term has exponents ( (1, 1, ldots, 1, 0) ). So, the exponents are all 1s except for one 0. Comparing these two exponent sequences: on the left, each term has one ( n-1 ) and the rest 0s, and on the right, each term has ( n-1 ) ones and one 0. Is one of these sequences majorizing the other? Let's check.For majorization, we need to arrange the exponents in non-increasing order and check if the partial sums of the first sequence are greater than or equal to those of the second.For the left-hand side, the exponents in non-increasing order are ( (n-1, 0, 0, ldots, 0) ).For the right-hand side, the exponents in non-increasing order are ( (1, 1, ldots, 1, 0) ).Now, let's compare the partial sums:- First term: ( n-1 ) vs. 1. Clearly, ( n-1 geq 1 ) for ( n geq 2 ).- Second term: ( n-1 + 0 = n-1 ) vs. ( 1 + 1 = 2 ). Again, ( n-1 geq 2 ) for ( n geq 3 ).Continuing this way, each partial sum of the left-hand side exponents is greater than or equal to the corresponding partial sum of the right-hand side exponents.Therefore, by Muirhead's inequality, the symmetric sum on the left is greater than or equal to the symmetric sum on the right.So, this would imply that:[sum_{i=1}^n x_i^{n-1} geq sum_{i=1}^n frac{1}{x_i}]Which is exactly what we needed to prove.Wait, but I should make sure that Muirhead's conditions are fully satisfied. Muirhead requires that the sequences are majorized, which we checked, and that the inequality is symmetric, which it is because all variables are treated equally.Also, since all ( x_i ) are positive, Muirhead's inequality applies without any issues.Therefore, the inequality holds by Muirhead's inequality.Alternatively, if I didn't recall Muirhead, maybe I could have used AM-GM in a different way. Let me see.Consider each term ( x_i^{n-1} ). By AM-GM, the arithmetic mean of ( n-1 ) copies of ( x_i ) and one copy of 1 would be greater than or equal to the geometric mean. Wait, but that might not directly relate to the reciprocal terms.Alternatively, perhaps using weighted AM-GM. But I think Muirhead is the more straightforward approach here.Another thought: since the product of all ( x_i ) is 1, maybe taking logarithms could help, but I don't see an immediate way to apply that here.Alternatively, consider homogenizing the inequality. Since the product is 1, we can express variables in terms of their logarithms or something else, but again, I think Muirhead is the key here.So, in conclusion, by recognizing that both sides are symmetric sums and that the exponent sequence on the left majorizes the one on the right, we can apply Muirhead's inequality to establish the desired result.</think>
❓:A trapezoid with its smaller base equal to 4 is inscribed in a circle with radius (sqrt{7}). Through a point on this circle, a tangent parallel to one of the trapezoid's non-parallel sides is drawn. A chord of length 5, parallel to the bases of the trapezoid, is also drawn through this point. Find the length of the trapezoid's diagonal and the area of the trapezoid.
🧠:<think>Okay, so I have this problem about a trapezoid inscribed in a circle. The trapezoid has a smaller base of 4 units, and the circle has a radius of √7. There's also a tangent line drawn through a point on the circle, which is parallel to one of the trapezoid's non-parallel sides. Additionally, there's a chord of length 5, parallel to the bases of the trapezoid, drawn through this same point. I need to find the length of the trapezoid's diagonal and its area.First, let me visualize this. A trapezoid inscribed in a circle means it's a cyclic trapezoid. I remember that in a cyclic trapezoid, the legs are equal, so it's an isosceles trapezoid. That means the non-parallel sides (the legs) are equal in length, and the base angles are equal.Given that the smaller base is 4, let's denote the trapezoid as ABCD with BC = 4 being the smaller base. The circle has a radius of √7, so the diameter is 2√7. The trapezoid is inscribed in this circle, so all four vertices lie on the circumference.Now, through a point on the circle, a tangent is drawn parallel to one of the non-parallel sides. Let's assume this tangent is parallel to side AB. Since the tangent is parallel to AB, the angle between the tangent and the radius at the point of contact should be equal to the angle between AB and the radius at point A or B.Also, through this same point, a chord of length 5 is drawn, which is parallel to the bases BC and AD. So, this chord is parallel to both bases and has a length of 5.I need to find the length of the diagonal and the area of the trapezoid.Let me recall that in a cyclic trapezoid, the sum of each pair of opposite angles is 180 degrees. Also, the diagonals are equal in length.Since the trapezoid is cyclic, it must be isosceles, so the legs AB and CD are equal. Let me denote the length of the legs as x.Given that the circle has a radius of √7, the diameter is 2√7. The maximum distance between any two points on the circle is the diameter, so the length of the diagonal cannot exceed 2√7. However, the chord of length 5 is drawn through a point, which is shorter than the diameter, so that's consistent.Let me consider the chord of length 5. Since it's parallel to the bases, it must be somewhere between the two bases. Let me denote the point where the tangent and the chord are drawn as point P on the circle.Since the tangent at P is parallel to AB, the angle between the tangent and the radius OP (where O is the center of the circle) is equal to the angle between AB and OA or OB.Let me denote the center of the circle as O. Since ABCD is cyclic, O is equidistant from all four vertices.Let me try to find the coordinates of the points to model this problem. Maybe placing the trapezoid in a coordinate system with the center at the origin.Let me assume that the trapezoid is symmetric with respect to the y-axis. So, let me place the smaller base BC on the x-axis, centered at the origin. So, point B is at (-2, 0) and point C is at (2, 0). Then, the larger base AD will be somewhere above the x-axis, symmetric about the y-axis.Let me denote the coordinates of A as (-a, h) and D as (a, h), where h is the height of the trapezoid, and a is half the length of the larger base AD.Since all four points lie on the circle with radius √7, the distance from each point to the center (0,0) must be √7.So, for point B (-2, 0), the distance is √[(-2)^2 + 0^2] = √4 = 2, which is less than √7. Wait, that can't be. If the radius is √7, then all points must be at a distance of √7 from the center. So, my initial placement is incorrect.Hmm, I need to adjust my coordinate system. Maybe I shouldn't place the smaller base BC on the x-axis. Instead, perhaps the trapezoid is rotated such that both bases are not aligned with the axes.Alternatively, maybe I can use trigonometric relationships.Let me consider the circle with center O. Let me denote the points A, B, C, D on the circumference. Since ABCD is an isosceles trapezoid, the arcs AB and CD are equal.Let me denote the measure of arc AB as θ, so arc CD is also θ. Then, the remaining arcs BC and AD must each be (360° - 2θ)/2 = 180° - θ.But since the trapezoid is cyclic, the sum of the measures of opposite angles is 180°, so angle ABC + angle ADC = 180°, and similarly for the other pair.Wait, maybe using central angles would help. The central angles corresponding to the sides of the trapezoid.Let me denote the central angles:- Let angle AOB = α- Angle BOC = β- Angle COD = γ- Angle DOA = δSince ABCD is cyclic, the sum of these angles is 360°, so α + β + γ + δ = 360°.Also, since ABCD is an isosceles trapezoid, sides AB and CD are equal, so their corresponding central angles should be equal. Therefore, α = γ.Similarly, sides BC and AD are the bases. Since BC is the smaller base, its central angle β is smaller than δ, the central angle for AD.So, we have α = γ, and β + δ = 360° - 2α.Given that BC = 4, which is the chord length corresponding to central angle β. The chord length formula is 2R sin(θ/2), where θ is the central angle.So, for chord BC: 4 = 2√7 sin(β/2). Therefore, sin(β/2) = 4 / (2√7) = 2/√7.Similarly, for chord AD, which is the larger base, its length is 2√7 sin(δ/2). Let me denote the length of AD as y, so y = 2√7 sin(δ/2).Also, the legs AB and CD have length x = 2√7 sin(α/2).Now, since the trapezoid is isosceles, the legs are equal, and the non-parallel sides make equal angles with the bases.Additionally, there is a chord of length 5 through point P, which is parallel to the bases. Let me denote this chord as PQ, where Q is another point on the circle. Since PQ is parallel to BC and AD, it must be another chord of the circle, parallel to the bases.Given that PQ is of length 5, which is between the lengths of BC (4) and AD (y). So, 4 < 5 < y, assuming y > 5.Wait, but actually, depending on where the chord is drawn, it could be between BC and AD or beyond AD. But since it's drawn through a point on the circle, it's likely between BC and AD.The chord PQ is parallel to BC and AD, so it must subtend the same central angle as BC or AD. Wait, no, because it's a different chord. The central angle for PQ would depend on its distance from the center.Wait, maybe I can relate the length of chord PQ to its distance from the center.The formula for the length of a chord is 2√(R² - d²), where d is the distance from the center to the chord.Given that PQ has length 5, so 5 = 2√( (√7)² - d² ) = 2√(7 - d²). Therefore, √(7 - d²) = 5/2, so 7 - d² = 25/4, which gives d² = 7 - 25/4 = (28 - 25)/4 = 3/4. Therefore, d = √(3)/2.So, the distance from the center to chord PQ is √3/2.Now, since PQ is parallel to BC and AD, and BC is at a certain distance from the center, and AD is at another distance.Wait, but I don't know the distances of BC and AD from the center yet.Alternatively, since PQ is parallel to BC and AD, and it's drawn through point P, which is on the circle, the tangent at P is parallel to one of the non-parallel sides, say AB.So, the tangent at P is parallel to AB. Since AB is a side of the trapezoid, which is a chord of the circle. The tangent at P is parallel to AB, so the angle between the tangent and the radius OP is equal to the angle between AB and OA.Wait, the angle between a tangent and a chord is equal to the angle in the alternate segment. So, the angle between tangent at P and chord PQ is equal to the angle that chord PQ makes with the alternate segment.But since the tangent is parallel to AB, the angle between tangent at P and PQ is equal to the angle between AB and PQ.But PQ is parallel to BC and AD, so the angle between AB and PQ is equal to the angle between AB and BC, which is one of the base angles of the trapezoid.This might be getting too abstract. Maybe I should try to relate the central angles.Given that the chord PQ has length 5 and is at a distance of √3/2 from the center, as calculated earlier.Now, let me consider the position of point P. Since the tangent at P is parallel to AB, and AB is a chord, the angle between tangent at P and radius OP is equal to the angle between AB and OA.Let me denote the angle between AB and OA as θ. Then, the angle between tangent at P and OP is also θ.But the tangent at P is parallel to AB, so the angle between tangent at P and PQ (which is parallel to BC) is equal to the angle between AB and BC, which is the base angle of the trapezoid.Wait, maybe I can find the central angles corresponding to BC and AD.Given that BC = 4, which corresponds to central angle β, so 4 = 2√7 sin(β/2), so sin(β/2) = 2/√7.Similarly, chord PQ = 5 corresponds to central angle φ, so 5 = 2√7 sin(φ/2), so sin(φ/2) = 5/(2√7).But wait, sin(φ/2) = 5/(2√7) ≈ 5/(4.899) ≈ 1.02, which is greater than 1, which is impossible. That can't be.Wait, that suggests that my assumption is wrong. Maybe chord PQ is not subtended by central angle φ, but rather, since it's parallel to BC and AD, it's actually a translated version, so its central angle is the same as BC or AD.Wait, no, because it's a different chord. The central angle would depend on its position.Wait, perhaps I made a mistake in calculating the distance from the center to chord PQ.Let me recalculate:Chord length = 2√(R² - d²)Given chord length = 5, R = √7.So, 5 = 2√(7 - d²)Divide both sides by 2: 5/2 = √(7 - d²)Square both sides: 25/4 = 7 - d²So, d² = 7 - 25/4 = (28 - 25)/4 = 3/4Thus, d = √(3)/2So, the distance from the center to chord PQ is √3/2.Now, since PQ is parallel to BC and AD, and the distance from the center to PQ is √3/2, we can relate this to the distances from the center to BC and AD.Let me denote the distance from the center to BC as h1 and to AD as h2.Since BC is a chord of length 4, its distance from the center is h1.Using the chord length formula: 4 = 2√(7 - h1²)So, 2 = √(7 - h1²)Square both sides: 4 = 7 - h1²Thus, h1² = 3, so h1 = √3.Similarly, for chord AD of length y, its distance from the center is h2.So, y = 2√(7 - h2²)Now, since PQ is between BC and AD, its distance from the center is √3/2, which is less than h1 = √3, so PQ is closer to the center than BC.Wait, but if BC is at distance √3 from the center, and PQ is at distance √3/2, which is closer, so PQ is between the center and BC.But in the trapezoid, the bases BC and AD are on opposite sides of the center, right? Because in a cyclic trapezoid, the bases are on opposite sides of the center.Wait, no, actually, in a cyclic trapezoid, the bases are on the same side of the center if it's isosceles. Wait, no, that doesn't make sense.Wait, in an isosceles trapezoid inscribed in a circle, the bases must be symmetric with respect to the center. So, the center lies on the line of symmetry, which is the perpendicular bisector of both bases.Therefore, the distances from the center to each base are equal in magnitude but opposite in direction. So, if BC is at distance h1 above the center, AD is at distance h1 below the center, or vice versa.Wait, but in that case, the distance from the center to BC is h1, and to AD is also h1, but in opposite directions.But earlier, I found that the distance from the center to BC is √3, so h1 = √3.Similarly, the distance from the center to AD would also be √3, but in the opposite direction.But then, the chord PQ is at distance √3/2 from the center, which is between the center and BC.Wait, but if BC is at distance √3 from the center, and PQ is at √3/2, which is closer to the center, then PQ is between the center and BC.But in the trapezoid, the bases BC and AD are on opposite sides of the center, so PQ is on the same side as BC but closer to the center.Wait, but then PQ is not part of the trapezoid, it's another chord through point P.Wait, maybe I need to consider the position of point P.Since the tangent at P is parallel to AB, which is a leg of the trapezoid, and PQ is a chord through P parallel to the bases.So, point P is on the circle, and the tangent at P is parallel to AB, which is a leg. So, the tangent at P is parallel to AB, meaning that the angle between OP (radius) and the tangent is equal to the angle between OA and AB.Since AB is a chord, the angle between OA and AB is equal to half the central angle subtended by AB.Wait, maybe I can use the property that the angle between a tangent and a chord is equal to the angle in the alternate segment.So, the angle between tangent at P and chord PQ is equal to the angle that chord PQ makes with the alternate segment.But since the tangent is parallel to AB, the angle between tangent at P and PQ is equal to the angle between AB and PQ.But PQ is parallel to BC, so the angle between AB and PQ is equal to the angle between AB and BC, which is the base angle of the trapezoid.Let me denote the base angle at B as φ. Then, the angle between AB and BC is φ.Therefore, the angle between tangent at P and PQ is φ.But the angle between tangent at P and PQ is also equal to the angle in the alternate segment, which is the angle between PQ and the arc opposite to it.Wait, maybe this is getting too convoluted.Alternatively, since the tangent at P is parallel to AB, the angle between OP and AB is equal to the angle between OP and the tangent, which is 90 degrees minus the angle between OP and PQ.Wait, no, the tangent at P is perpendicular to OP, so the angle between OP and the tangent is 90 degrees.Wait, no, the tangent at P is parallel to AB, so the angle between OP and AB is equal to the angle between OP and the tangent, which is 90 degrees.Wait, no, the tangent at P is parallel to AB, so the direction of AB is the same as the direction of the tangent at P. Therefore, the angle between OP and AB is equal to the angle between OP and the tangent, which is 90 degrees.Wait, that can't be, because AB is a chord, not a tangent.Wait, perhaps I need to consider the slope of AB and the slope of the tangent at P.Alternatively, maybe I can use coordinates.Let me try to place the trapezoid in a coordinate system.Let me assume the center O is at (0,0). Let me denote point B as (x1, y1), point C as (x2, y2), point A as (x3, y3), and point D as (x4, y4).Since ABCD is an isosceles trapezoid, it's symmetric about the y-axis. So, points B and C are symmetric with respect to the y-axis, as are points A and D.So, let me denote point B as (-a, b), point C as (a, b), point A as (-c, d), and point D as (c, d).Since ABCD is cyclic, all four points lie on the circle of radius √7 centered at (0,0). Therefore, for each point, the distance from the origin is √7.So, for point B: (-a)^2 + b^2 = 7 => a² + b² = 7Similarly, for point A: (-c)^2 + d^2 = 7 => c² + d² = 7Now, the smaller base BC has length 4. Since B is (-a, b) and C is (a, b), the distance between B and C is 2a = 4, so a = 2.Therefore, from point B: a² + b² = 7 => 4 + b² = 7 => b² = 3 => b = √3 or -√3. Let's assume b = √3 for simplicity, so point B is (-2, √3) and point C is (2, √3).Now, the larger base AD has length y, which is 2c, since A is (-c, d) and D is (c, d). So, the length of AD is 2c.The legs AB and CD have length x, which can be calculated using the distance formula between points A and B, and points C and D.Distance AB: sqrt[(-c + 2)^2 + (d - √3)^2] = xSimilarly, distance CD: sqrt[(c - 2)^2 + (d - √3)^2] = xSince AB = CD, which is consistent.Now, the height of the trapezoid is the vertical distance between the two bases BC and AD. Since BC is at y = √3 and AD is at y = d, the height h is |d - √3|.Now, the area of the trapezoid is (1/2)*(BC + AD)*h = (1/2)*(4 + 2c)*|d - √3|.We need to find c and d.Also, since point P is on the circle, and through P, a tangent is drawn parallel to AB, and a chord PQ of length 5 is drawn parallel to BC and AD.Let me denote point P as (p, q), lying on the circle, so p² + q² = 7.The tangent at P is parallel to AB. The slope of AB is (d - √3)/( -c + 2). The slope of the tangent at P is equal to the slope of AB.But the slope of the tangent at P is given by the derivative at that point. For a circle centered at (0,0), the slope of the tangent at (p, q) is -p/q.Therefore, -p/q = (d - √3)/(-c + 2)Simplify: -p/q = (d - √3)/(-c + 2) => p/q = (d - √3)/(c - 2)So, p/q = (d - √3)/(c - 2) => p = q*(d - √3)/(c - 2)Also, since chord PQ is parallel to BC and AD, which are horizontal lines (since BC is from (-2, √3) to (2, √3), so it's horizontal). Therefore, chord PQ is also horizontal, so it has the same y-coordinate as point P.Wait, no, chord PQ is parallel to BC and AD, which are horizontal, so PQ is horizontal. Therefore, chord PQ is a horizontal chord passing through point P, so it has the same y-coordinate as P, which is q.But the length of chord PQ is 5. The length of a horizontal chord at height q is 2√(7 - q²). So, 2√(7 - q²) = 5 => √(7 - q²) = 5/2 => 7 - q² = 25/4 => q² = 7 - 25/4 = (28 - 25)/4 = 3/4 => q = ±√(3)/2.Therefore, point P has coordinates (p, q) where q = √3/2 or q = -√3/2.But since the tangent at P is parallel to AB, which is a leg of the trapezoid, and AB is above BC, which is at y = √3, I think point P is above BC, so q = √3/2.Wait, but BC is at y = √3, and the center is at (0,0). So, if point P is above BC, it would be at y > √3, but √3 ≈ 1.732, and √3/2 ≈ 0.866, which is below BC. So, maybe point P is below BC.Wait, but the chord PQ is parallel to BC and AD, and has length 5. If BC is at y = √3, and AD is at y = d, then PQ could be between them or outside.But since the length of PQ is 5, which is longer than BC (4), it must be closer to the center. Wait, no, the length of a chord increases as it approaches the center. So, a chord of length 5 is longer than BC (4), so it's closer to the center.Given that BC is at distance h1 = √3 from the center, and PQ is at distance √3/2, which is closer to the center, so PQ is between the center and BC.Therefore, point P is on the same side as BC but closer to the center, so its y-coordinate is q = √3/2.So, point P is (p, √3/2), and p² + (√3/2)² = 7 => p² + 3/4 = 7 => p² = 7 - 3/4 = 25/4 => p = ±5/2.Therefore, point P is either (5/2, √3/2) or (-5/2, √3/2).Now, the tangent at P is parallel to AB. The slope of the tangent at P is -p/q = -(5/2)/(√3/2) = -5/√3.The slope of AB is (d - √3)/(-c + 2). So, setting them equal:-5/√3 = (d - √3)/(-c + 2)Multiply both sides by (-c + 2):-5/√3 * (-c + 2) = d - √3Simplify:(5/√3)(c - 2) = d - √3So, d = (5/√3)(c - 2) + √3Now, we also know that point A is (-c, d) and lies on the circle, so (-c)^2 + d^2 = 7 => c² + d² = 7.Substitute d from above:c² + [(5/√3)(c - 2) + √3]^2 = 7Let me expand this:First, let me write d as:d = (5/√3)(c - 2) + √3 = (5c - 10)/√3 + √3 = (5c - 10 + 3)/√3 = (5c - 7)/√3Wait, no, that's incorrect. Let me correctly expand:d = (5/√3)(c - 2) + √3Let me write it as:d = (5c - 10)/√3 + √3 = (5c - 10 + 3)/√3 = (5c - 7)/√3Wait, no, that's not correct. When adding fractions, I need a common denominator.Let me write √3 as (√3 * √3)/√3 = 3/√3.So, d = (5c - 10)/√3 + 3/√3 = (5c - 10 + 3)/√3 = (5c - 7)/√3Yes, that's correct.So, d = (5c - 7)/√3Now, substitute into c² + d² = 7:c² + [(5c - 7)/√3]^2 = 7Simplify:c² + (25c² - 70c + 49)/3 = 7Multiply both sides by 3 to eliminate the denominator:3c² + 25c² - 70c + 49 = 21Combine like terms:28c² - 70c + 49 - 21 = 0 => 28c² - 70c + 28 = 0Divide all terms by 14:2c² - 5c + 2 = 0Now, solve for c:Using quadratic formula: c = [5 ± √(25 - 16)] / 4 = [5 ± 3]/4So, c = (5 + 3)/4 = 8/4 = 2, or c = (5 - 3)/4 = 2/4 = 1/2Now, c = 2 would mean that point A is (-2, d), but point B is (-2, √3). Since ABCD is a trapezoid, points A and B must be distinct, so c cannot be 2. Therefore, c = 1/2.So, c = 1/2Now, find d:d = (5c - 7)/√3 = (5*(1/2) - 7)/√3 = (5/2 - 7)/√3 = (-9/2)/√3 = (-9)/(2√3) = (-3√3)/2So, d = -3√3/2Therefore, point A is (-1/2, -3√3/2) and point D is (1/2, -3√3/2)Now, let's verify if this makes sense.The distance from A to B:Points A (-1/2, -3√3/2) and B (-2, √3)Distance AB: sqrt[(-1/2 + 2)^2 + (-3√3/2 - √3)^2] = sqrt[(3/2)^2 + (-5√3/2)^2] = sqrt[9/4 + 75/4] = sqrt[84/4] = sqrt[21] ≈ 4.583Similarly, distance CD should be the same.Points C (2, √3) and D (1/2, -3√3/2)Distance CD: sqrt[(2 - 1/2)^2 + (√3 + 3√3/2)^2] = sqrt[(3/2)^2 + (5√3/2)^2] = sqrt[9/4 + 75/4] = sqrt[84/4] = sqrt[21] ≈ 4.583Good, so AB = CD = sqrt(21)Now, the larger base AD has length 2c = 2*(1/2) = 1? Wait, that can't be, because AD is supposed to be the larger base, longer than BC which is 4.Wait, that's a problem. Because if c = 1/2, then AD = 2c = 1, which is shorter than BC = 4. That contradicts the fact that AD is the larger base.Wait, so I must have made a mistake in my calculations.Let me go back.When I solved for c, I got c = 2 or c = 1/2. I discarded c = 2 because it would coincide with point B, but maybe that's not the case.Wait, if c = 2, then point A would be (-2, d). Let's compute d in that case.d = (5c - 7)/√3 = (10 - 7)/√3 = 3/√3 = √3So, point A would be (-2, √3), which is the same as point B. That's not possible, so c = 2 is invalid.Therefore, c = 1/2 is the only solution, but that leads to AD = 1, which is shorter than BC = 4, which contradicts the trapezoid being isosceles with BC as the smaller base.Therefore, I must have made an error in my earlier steps.Let me check where I might have gone wrong.When I calculated d, I had:d = (5c - 7)/√3But let me re-examine the earlier step.We had:d = (5/√3)(c - 2) + √3Let me compute this correctly:d = (5/√3)(c - 2) + √3= (5c - 10)/√3 + √3To combine these, I need a common denominator:= (5c - 10)/√3 + (√3 * √3)/√3= (5c - 10 + 3)/√3= (5c - 7)/√3Yes, that's correct.Then, substituting into c² + d² = 7:c² + [(5c - 7)/√3]^2 = 7= c² + (25c² - 70c + 49)/3 = 7Multiply by 3:3c² + 25c² - 70c + 49 = 21Combine:28c² - 70c + 28 = 0Divide by 14:2c² - 5c + 2 = 0Solutions: c = [5 ± √(25 - 16)] / 4 = [5 ± 3]/4 => c = 2 or c = 1/2So, that's correct.But as we saw, c = 1/2 leads to AD = 1, which is too short.Wait, maybe I made a mistake in assuming the position of point P.Earlier, I assumed that point P is above BC, but since the chord PQ is at y = √3/2, which is below BC at y = √3, maybe point P is below BC.Wait, but if point P is below BC, then its y-coordinate is √3/2, which is still above the center at y=0.Wait, but in that case, the tangent at P is parallel to AB, which is a leg going from B (-2, √3) to A (-1/2, -3√3/2). So, the slope of AB is (d - √3)/(-c + 2) = (-3√3/2 - √3)/(-1/2 + 2) = (-5√3/2)/(3/2) = (-5√3)/3So, the slope of AB is -5√3/3The slope of the tangent at P is -p/qSince point P is (5/2, √3/2), the slope is - (5/2)/(√3/2) = -5/√3 = -5√3/3, which matches the slope of AB.So, that's consistent.But then, point A is at (-1/2, -3√3/2), which is below the center, and point D is at (1/2, -3√3/2). So, the larger base AD is at y = -3√3/2, which is below the center.Wait, but then the distance from the center to AD is |d| = 3√3/2, which is greater than the distance to BC, which is √3.But in a trapezoid, the larger base should be further from the center if it's on the opposite side.Wait, but in this case, BC is at y = √3, and AD is at y = -3√3/2, which is further from the center than BC.Wait, but the distance from the center to BC is √3, and to AD is 3√3/2, which is indeed further.But the length of BC is 4, and the length of AD is 1, which is shorter, which contradicts the fact that AD should be the larger base.Wait, so perhaps I have the labeling wrong. Maybe BC is the larger base, and AD is the smaller base.But the problem states that the smaller base is 4, so BC = 4 is the smaller base, so AD should be the larger base, longer than 4.But in my calculation, AD is only 1, which is shorter. So, something is wrong.Wait, perhaps I made a mistake in the sign of d.When I solved for d, I got d = (5c - 7)/√3. For c = 1/2, d = (5*(1/2) - 7)/√3 = (2.5 - 7)/√3 = (-4.5)/√3 = -3√3/2.But if I take d = 3√3/2 instead, would that make sense?Wait, no, because when I solved for d, it was d = (5c - 7)/√3, which for c = 1/2 gives d = -3√3/2.Alternatively, maybe I should have considered the other point P at (-5/2, √3/2). Let me try that.If point P is (-5/2, √3/2), then the slope of the tangent at P is -p/q = -(-5/2)/(√3/2) = 5/√3.The slope of AB is (d - √3)/(-c + 2). Setting them equal:5/√3 = (d - √3)/(-c + 2)Multiply both sides by (-c + 2):5/√3 * (-c + 2) = d - √3So, d = 5/√3 * (-c + 2) + √3= (-5c + 10)/√3 + √3= (-5c + 10 + 3)/√3= (-5c + 13)/√3Now, substitute into c² + d² = 7:c² + [(-5c + 13)/√3]^2 = 7= c² + (25c² - 130c + 169)/3 = 7Multiply by 3:3c² + 25c² - 130c + 169 = 21Combine:28c² - 130c + 148 = 0Divide by 2:14c² - 65c + 74 = 0Discriminant: 65² - 4*14*74 = 4225 - 4144 = 81Solutions: c = [65 ± 9]/28So, c = (65 + 9)/28 = 74/28 = 37/14 ≈ 2.643Or c = (65 - 9)/28 = 56/28 = 2Again, c = 2 would make point A coincide with B, so invalid. So, c = 37/14Now, compute d:d = (-5c + 13)/√3 = (-5*(37/14) + 13)/√3 = (-185/14 + 182/14)/√3 = (-3/14)/√3 = (-3)/(14√3) = (-√3)/14So, d = -√3/14Therefore, point A is (-37/14, -√3/14) and point D is (37/14, -√3/14)Now, compute the length of AD:AD = 2c = 2*(37/14) = 37/7 ≈ 5.2857, which is longer than BC = 4, so that makes sense.Now, compute the distance AB:Points A (-37/14, -√3/14) and B (-2, √3)Distance AB: sqrt[(-37/14 + 2)^2 + (-√3/14 - √3)^2]First, compute the x-coordinate difference:-37/14 + 2 = -37/14 + 28/14 = (-37 + 28)/14 = (-9)/14Y-coordinate difference:-√3/14 - √3 = -√3/14 - 14√3/14 = (-15√3)/14So, distance AB:sqrt[(-9/14)^2 + (-15√3/14)^2] = sqrt[(81/196) + (225*3)/196] = sqrt[(81 + 675)/196] = sqrt[756/196] = sqrt[3.857] ≈ 1.964Wait, that's much shorter than BC = 4, which doesn't make sense because in a trapezoid, the legs are usually longer than the difference of the bases.Wait, maybe I made a mistake in calculations.Wait, let me recalculate the distance AB.Point A: (-37/14, -√3/14)Point B: (-2, √3) = (-28/14, 14√3/14)So, x difference: (-37/14 + 28/14) = (-9)/14Y difference: (-√3/14 - 14√3/14) = (-15√3)/14So, distance AB: sqrt[(-9/14)^2 + (-15√3/14)^2] = sqrt[(81/196) + (225*3)/196] = sqrt[(81 + 675)/196] = sqrt[756/196] = sqrt[3.857] ≈ 1.964Hmm, that seems too short. Maybe I made a mistake in the value of c.Wait, c = 37/14 ≈ 2.643, so point A is (-2.643, -0.129), and point B is (-2, 1.732). The distance between them is approximately sqrt[(0.643)^2 + (1.861)^2] ≈ sqrt[0.413 + 3.464] ≈ sqrt[3.877] ≈ 1.969, which matches the earlier calculation.But in a trapezoid, the legs should be longer than the difference of the bases. The difference between the bases is AD - BC = 37/7 - 4 = 37/7 - 28/7 = 9/7 ≈ 1.285. So, the legs being ~1.964 is longer than 1.285, which is acceptable.But let's check the height of the trapezoid.The height is the vertical distance between the two bases BC and AD.Point B is at y = √3 ≈ 1.732, and point A is at y = -√3/14 ≈ -0.129.So, the height h = √3 - (-√3/14) = √3 + √3/14 = (14√3 + √3)/14 = 15√3/14 ≈ 1.837Now, the area of the trapezoid is (1/2)*(BC + AD)*h = (1/2)*(4 + 37/7)*(15√3/14)First, compute 4 + 37/7 = 28/7 + 37/7 = 65/7So, area = (1/2)*(65/7)*(15√3/14) = (65/14)*(15√3/14) = (975√3)/196 ≈ 8.45Now, let's find the length of the diagonal.The diagonals of a trapezoid are equal in length. Let's compute the distance between A and C.Point A: (-37/14, -√3/14)Point C: (2, √3) = (28/14, 14√3/14)Distance AC: sqrt[(28/14 + 37/14)^2 + (14√3/14 + √3/14)^2] = sqrt[(65/14)^2 + (15√3/14)^2] = sqrt[(4225/196) + (225*3)/196] = sqrt[(4225 + 675)/196] = sqrt[4900/196] = sqrt[25] = 5So, the length of the diagonal AC is 5.Therefore, the diagonal is 5, and the area is 975√3/196.But let me check if this makes sense.Given that the circle has radius √7, the maximum distance between two points is 2√7 ≈ 5.2915. So, a diagonal of 5 is possible.Also, the area seems reasonable.So, after correcting the position of point P and recalculating, I find that the diagonal is 5 and the area is 975√3/196.Therefore, the final answers are:Length of the diagonal: 5Area of the trapezoid: 975√3/196</think>
❓:Let x_{i} geq 0 (i=1,2,3, cdots, n) and sum_{i=1}^{n} x_{i}^{2}+2 sum_{1 leq k<j leq n} sqrt{frac{k}{j}} x_{k} x_{j}=1. Find the maximum and minimum values of sum_{i=1}^{n} x_{i}.
🧠:<think>Okay, so I have this problem where I need to find the maximum and minimum values of the sum of variables ( x_1, x_2, ldots, x_n ) given a certain condition. The condition is a bit complicated: it's the sum of the squares of each ( x_i ) plus twice the sum of the products of ( x_k ) and ( x_j ) multiplied by the square root of ( frac{k}{j} ) for all ( k < j ). And all the ( x_i ) are non-negative. First, let me write down the given condition to make it clearer:[sum_{i=1}^{n} x_{i}^{2} + 2 sum_{1 leq k < j leq n} sqrt{frac{k}{j}} x_{k} x_{j} = 1]And I need to find the maximum and minimum of:[sum_{i=1}^{n} x_{i}]Hmm. So, the expression on the left side of the equation looks a bit like a quadratic form. Maybe I can rewrite it in a more familiar way. Let me think about how to express this as a square of a sum or something similar.Wait, if I consider the square of the sum ( left( sum_{i=1}^{n} a_i x_i right)^2 ), it would expand to:[sum_{i=1}^{n} a_i^2 x_i^2 + 2 sum_{1 leq k < j leq n} a_k a_j x_k x_j]Comparing this with the given condition, I notice that if I set ( a_i^2 = 1 ) for all ( i ), then the first term would match. But the cross terms in the given condition have coefficients ( sqrt{frac{k}{j}} ), whereas in the square of the sum, the coefficients are ( a_k a_j ). So, if I can find ( a_i ) such that ( a_k a_j = sqrt{frac{k}{j}} ), then the cross terms would match as well.Let me see. If ( a_k a_j = sqrt{frac{k}{j}} ), then perhaps ( a_i = sqrt{frac{c}{i}} ) for some constant ( c ). Let me test this:If ( a_i = sqrt{frac{c}{i}} ), then ( a_k a_j = sqrt{frac{c}{k}} sqrt{frac{c}{j}} = frac{c}{sqrt{kj}} ). But in the given condition, the cross term is ( sqrt{frac{k}{j}} ). So, if I set ( frac{c}{sqrt{kj}} = sqrt{frac{k}{j}} ), then ( c = frac{k}{sqrt{kj}} times sqrt{kj} )... Wait, that seems a bit convoluted. Maybe I need a different approach.Alternatively, perhaps I can factor the given expression. Let me think about whether the given quadratic form can be expressed as a square of a linear combination of ( x_i ). If so, then the condition would be the square equals 1, which would make the sum of ( x_i ) related to that square.Wait, another thought: if I let ( y_i = sqrt{i} x_i ), then maybe the given condition can be rewritten in terms of ( y_i ). Let me try that substitution.Let ( y_i = sqrt{i} x_i ). Then ( x_i = frac{y_i}{sqrt{i}} ). Substitute into the given condition:First term: ( sum_{i=1}^{n} x_i^2 = sum_{i=1}^{n} frac{y_i^2}{i} )Second term: ( 2 sum_{1 leq k < j leq n} sqrt{frac{k}{j}} x_k x_j = 2 sum_{1 leq k < j leq n} sqrt{frac{k}{j}} cdot frac{y_k}{sqrt{k}} cdot frac{y_j}{sqrt{j}} )Simplify the second term:( 2 sum_{1 leq k < j leq n} sqrt{frac{k}{j}} cdot frac{y_k}{sqrt{k}} cdot frac{y_j}{sqrt{j}} = 2 sum_{1 leq k < j leq n} frac{sqrt{k}}{sqrt{j}} cdot frac{y_k}{sqrt{k}} cdot frac{y_j}{sqrt{j}} = 2 sum_{1 leq k < j leq n} frac{y_k y_j}{j} )Wait, that doesn't seem right. Let me recast it:( sqrt{frac{k}{j}} cdot frac{1}{sqrt{k}} cdot frac{1}{sqrt{j}} = frac{sqrt{k}}{sqrt{j}} cdot frac{1}{sqrt{k}} cdot frac{1}{sqrt{j}} = frac{1}{j} )So, the second term becomes ( 2 sum_{1 leq k < j leq n} frac{y_k y_j}{j} ). Hmm, that seems a bit messy. Maybe this substitution isn't helpful.Wait, another approach: perhaps the given quadratic form is positive definite, so I can use the Cauchy-Schwarz inequality or something similar to relate the sum ( sum x_i ) to the given condition.Alternatively, maybe I can consider the given condition as a norm. If I think of the quadratic form as a norm squared, then the sum ( sum x_i ) is like a linear functional, and I can use the Cauchy-Schwarz inequality to bound it.Let me define a vector ( mathbf{x} = (x_1, x_2, ldots, x_n) ). Then, the given condition is ( mathbf{x}^T A mathbf{x} = 1 ), where ( A ) is a matrix with diagonal entries 1 and off-diagonal entries ( sqrt{frac{k}{j}} ) for ( k < j ).Wait, actually, the quadratic form is:[sum_{i=1}^{n} x_i^2 + 2 sum_{1 leq k < j leq n} sqrt{frac{k}{j}} x_k x_j]So, the matrix ( A ) would have ( A_{i,i} = 1 ) and ( A_{k,j} = sqrt{frac{k}{j}} ) for ( k < j ), and ( A_{j,k} = sqrt{frac{k}{j}} ) for ( j > k ). So, it's a symmetric matrix.Now, the quadratic form is ( mathbf{x}^T A mathbf{x} = 1 ). I need to find the maximum and minimum of ( mathbf{1}^T mathbf{x} ), where ( mathbf{1} ) is the vector of ones.This is a constrained optimization problem. To find the extrema of ( mathbf{1}^T mathbf{x} ) subject to ( mathbf{x}^T A mathbf{x} = 1 ) and ( x_i geq 0 ).Since all ( x_i ) are non-negative, we can consider the problem in the positive orthant.To solve this, I can use the method of Lagrange multipliers. Let me set up the Lagrangian:[mathcal{L} = sum_{i=1}^{n} x_i - lambda left( sum_{i=1}^{n} x_i^2 + 2 sum_{1 leq k < j leq n} sqrt{frac{k}{j}} x_k x_j - 1 right)]Taking partial derivatives with respect to each ( x_i ) and setting them to zero:For each ( i ):[frac{partial mathcal{L}}{partial x_i} = 1 - lambda left( 2x_i + 2 sum_{j neq i} sqrt{frac{min(i,j)}{max(i,j)}} x_j right) = 0]Wait, let me clarify the cross terms. For each ( x_i ), the derivative will involve the terms where ( x_i ) is multiplied by ( x_j ) for ( j neq i ). So, for each ( j > i ), the term is ( 2 sqrt{frac{i}{j}} x_j ), and for each ( j < i ), the term is ( 2 sqrt{frac{j}{i}} x_j ).Therefore, the derivative is:[1 - lambda left( 2x_i + 2 sum_{j=1}^{i-1} sqrt{frac{j}{i}} x_j + 2 sum_{j=i+1}^{n} sqrt{frac{i}{j}} x_j right) = 0]This gives us a system of equations:For each ( i ):[1 = 2 lambda x_i + 2 lambda sum_{j=1}^{i-1} sqrt{frac{j}{i}} x_j + 2 lambda sum_{j=i+1}^{n} sqrt{frac{i}{j}} x_j]Hmm, this seems quite involved. Maybe there's a pattern or a way to express this more simply.Alternatively, perhaps I can consider the vector ( mathbf{x} ) such that ( A mathbf{x} = mu mathbf{1} ), where ( mu ) is a scalar. Then, the maximum of ( mathbf{1}^T mathbf{x} ) would be related to the smallest eigenvalue or something like that.Wait, but since ( A ) is symmetric, the maximum of ( mathbf{1}^T mathbf{x} ) subject to ( mathbf{x}^T A mathbf{x} = 1 ) would be the square root of the largest eigenvalue of ( A^{-1} ) times the vector ( mathbf{1} ). Hmm, not sure.Alternatively, maybe I can think of this as a generalized eigenvalue problem. Let me denote ( mathbf{1}^T mathbf{x} = c ), and we have ( mathbf{x}^T A mathbf{x} = 1 ). To maximize ( c ), we can set up the ratio ( frac{c^2}{mathbf{x}^T A mathbf{x}} ) and find its maximum, which would be the largest eigenvalue of ( A^{-1} mathbf{1} mathbf{1}^T ), but I'm not sure.This seems complicated. Maybe I should try a simpler case first, like when ( n = 2 ), to see if I can spot a pattern.Let me try ( n = 2 ):Given:[x_1^2 + x_2^2 + 2 sqrt{frac{1}{2}} x_1 x_2 = 1]And we need to find the maximum and minimum of ( x_1 + x_2 ).Let me denote ( s = x_1 + x_2 ). Then, the given condition is:[x_1^2 + x_2^2 + 2 cdot frac{1}{sqrt{2}} x_1 x_2 = 1]Note that ( x_1^2 + x_2^2 = (x_1 + x_2)^2 - 2 x_1 x_2 = s^2 - 2 x_1 x_2 ). So, substituting:[s^2 - 2 x_1 x_2 + 2 cdot frac{1}{sqrt{2}} x_1 x_2 = 1]Simplify:[s^2 - 2 x_1 x_2 + sqrt{2} x_1 x_2 = 1][s^2 + (-2 + sqrt{2}) x_1 x_2 = 1]But I don't know ( x_1 x_2 ). Maybe I can express ( x_1 x_2 ) in terms of ( s ). Since ( x_1 + x_2 = s ), the maximum of ( x_1 x_2 ) is ( frac{s^2}{4} ) (by AM-GM inequality), but I don't know if that helps here.Alternatively, let me consider the given quadratic form as a quadratic in ( x_1 ) and ( x_2 ). Maybe I can diagonalize it or find its eigenvalues.The quadratic form is:[x_1^2 + x_2^2 + sqrt{2} x_1 x_2]The matrix ( A ) is:[A = begin{pmatrix}1 & frac{sqrt{2}}{2} frac{sqrt{2}}{2} & 1end{pmatrix}]The eigenvalues of this matrix can be found by solving ( det(A - lambda I) = 0 ):[det begin{pmatrix}1 - lambda & frac{sqrt{2}}{2} frac{sqrt{2}}{2} & 1 - lambdaend{pmatrix} = (1 - lambda)^2 - frac{1}{2} = 0][(1 - lambda)^2 = frac{1}{2}][1 - lambda = pm frac{sqrt{2}}{2}][lambda = 1 mp frac{sqrt{2}}{2}]So, the eigenvalues are ( 1 + frac{sqrt{2}}{2} ) and ( 1 - frac{sqrt{2}}{2} ).The maximum value of ( s = x_1 + x_2 ) subject to ( mathbf{x}^T A mathbf{x} = 1 ) would be related to the inverse of the smallest eigenvalue. Wait, actually, the maximum of ( mathbf{1}^T mathbf{x} ) is given by ( sqrt{mathbf{1}^T A^{-1} mathbf{1}} ).Let me compute ( A^{-1} ). Since ( A ) is symmetric, its inverse can be found using the eigenvalues. Alternatively, for a 2x2 matrix, the inverse is ( frac{1}{det A} begin{pmatrix} d & -b -c & a end{pmatrix} ).First, compute the determinant of ( A ):[det A = (1)(1) - left( frac{sqrt{2}}{2} right)^2 = 1 - frac{1}{2} = frac{1}{2}]So, ( A^{-1} = 2 begin{pmatrix} 1 & -frac{sqrt{2}}{2} -frac{sqrt{2}}{2} & 1 end{pmatrix} = begin{pmatrix} 2 & -sqrt{2} -sqrt{2} & 2 end{pmatrix} )Then, ( mathbf{1}^T A^{-1} mathbf{1} ) is:[begin{pmatrix} 1 & 1 end{pmatrix} begin{pmatrix} 2 & -sqrt{2} -sqrt{2} & 2 end{pmatrix} begin{pmatrix} 1 1 end{pmatrix}]Compute the product:First, multiply ( A^{-1} ) with ( mathbf{1} ):[begin{pmatrix} 2 & -sqrt{2} -sqrt{2} & 2 end{pmatrix} begin{pmatrix} 1 1 end{pmatrix} = begin{pmatrix} 2 - sqrt{2} -sqrt{2} + 2 end{pmatrix}]Then, take the dot product with ( mathbf{1} ):[(2 - sqrt{2}) + (-sqrt{2} + 2) = 4 - 2sqrt{2}]So, ( mathbf{1}^T A^{-1} mathbf{1} = 4 - 2sqrt{2} ). Therefore, the maximum value of ( s ) is ( sqrt{4 - 2sqrt{2}} ).Simplify ( sqrt{4 - 2sqrt{2}} ):Let me compute ( 4 - 2sqrt{2} approx 4 - 2.828 approx 1.172 ), so the square root is approximately 1.083.But let me see if it can be expressed more neatly. Let me assume ( sqrt{4 - 2sqrt{2}} = sqrt{a} - sqrt{b} ). Then, squaring both sides:[4 - 2sqrt{2} = a + b - 2sqrt{ab}]Comparing the terms:( a + b = 4 )( -2sqrt{ab} = -2sqrt{2} ) => ( sqrt{ab} = sqrt{2} ) => ( ab = 2 )So, solving ( a + b = 4 ) and ( ab = 2 ). The solutions are roots of ( t^2 - 4t + 2 = 0 ):[t = frac{4 pm sqrt{16 - 8}}{2} = frac{4 pm 2sqrt{2}}{2} = 2 pm sqrt{2}]So, ( a = 2 + sqrt{2} ), ( b = 2 - sqrt{2} ). Therefore,[sqrt{4 - 2sqrt{2}} = sqrt{2 + sqrt{2}} - sqrt{2 - sqrt{2}}]Wait, actually, let me check:If ( a = 2 + sqrt{2} ) and ( b = 2 - sqrt{2} ), then ( sqrt{a} - sqrt{b} = sqrt{2 + sqrt{2}} - sqrt{2 - sqrt{2}} ). Let me compute this:Compute ( sqrt{2 + sqrt{2}} approx sqrt{2 + 1.414} approx sqrt{3.414} approx 1.847 )Compute ( sqrt{2 - sqrt{2}} approx sqrt{2 - 1.414} approx sqrt{0.586} approx 0.765 )So, ( 1.847 - 0.765 approx 1.082 ), which matches the approximate value earlier. So, it's correct.But perhaps there's a better way to write ( sqrt{4 - 2sqrt{2}} ). Alternatively, maybe it's just fine as it is.So, for ( n = 2 ), the maximum of ( x_1 + x_2 ) is ( sqrt{4 - 2sqrt{2}} ).Now, what about the minimum? Since all ( x_i ) are non-negative, the minimum occurs when one of them is 1 and the others are 0. Because if we set ( x_1 = 1 ) and ( x_2 = 0 ), the condition becomes ( 1 + 0 + 0 = 1 ), which satisfies the condition, and the sum is 1. Similarly, setting ( x_2 = 1 ) and ( x_1 = 0 ) also gives a sum of 1. So, the minimum is 1.Wait, but in the case of ( n = 2 ), is 1 indeed the minimum? Let me check if there's a configuration where the sum is less than 1.Suppose both ( x_1 ) and ( x_2 ) are positive. Then, the quadratic form is ( x_1^2 + x_2^2 + sqrt{2} x_1 x_2 = 1 ). If I set ( x_1 = x_2 = t ), then:[2t^2 + sqrt{2} t^2 = 1 implies t^2 (2 + sqrt{2}) = 1 implies t = frac{1}{sqrt{2 + sqrt{2}}} approx 0.541]So, the sum ( x_1 + x_2 = 2t approx 1.082 ), which is greater than 1. So, indeed, the minimum is achieved when one variable is 1 and the others are 0.So, for ( n = 2 ), the minimum is 1 and the maximum is ( sqrt{4 - 2sqrt{2}} ).Hmm, interesting. Now, going back to the general case for ( n ). Maybe the minimum is always 1, achieved when one variable is 1 and the rest are 0. Let me check for ( n = 3 ).For ( n = 3 ), the condition is:[x_1^2 + x_2^2 + x_3^2 + 2 sqrt{frac{1}{2}} x_1 x_2 + 2 sqrt{frac{1}{3}} x_1 x_3 + 2 sqrt{frac{2}{3}} x_2 x_3 = 1]If I set ( x_1 = 1 ) and ( x_2 = x_3 = 0 ), the condition is satisfied, and the sum is 1. Similarly, setting ( x_2 = 1 ) or ( x_3 = 1 ) also gives a sum of 1. So, the minimum is indeed 1 for ( n = 3 ).What about the maximum? For ( n = 3 ), it's more complicated, but perhaps the maximum can be found similarly by considering the quadratic form and using eigenvalues or something else.But maybe there's a pattern here. For ( n = 2 ), the maximum is ( sqrt{4 - 2sqrt{2}} ), which is approximately 1.082. For ( n = 1 ), the maximum and minimum would both be 1, since there's only one variable.Wait, for ( n = 1 ), the condition is ( x_1^2 = 1 ), so ( x_1 = 1 ) (since ( x_1 geq 0 )), and the sum is 1. So, for ( n = 1 ), both max and min are 1.For ( n = 2 ), min is 1, max is ( sqrt{4 - 2sqrt{2}} approx 1.082 ).For ( n = 3 ), the maximum would be higher than for ( n = 2 ), I suppose.Wait, but let me think about the general case. Maybe the maximum is achieved when all variables are set in a certain way, perhaps proportional to something.Wait, another approach: since the quadratic form is given, maybe I can write it as ( (sum a_i x_i)^2 ) for some coefficients ( a_i ). If I can find such ( a_i ), then the condition becomes ( (sum a_i x_i)^2 = 1 ), and the sum ( sum x_i ) can be related to ( sum a_i x_i ) via Cauchy-Schwarz.Let me try to express the given quadratic form as a square. Suppose:[left( sum_{i=1}^{n} a_i x_i right)^2 = sum_{i=1}^{n} a_i^2 x_i^2 + 2 sum_{1 leq k < j leq n} a_k a_j x_k x_j]Comparing with the given condition:[sum_{i=1}^{n} x_i^2 + 2 sum_{1 leq k < j leq n} sqrt{frac{k}{j}} x_k x_j]So, we need:[a_i^2 = 1 quad text{for all } i]and[a_k a_j = sqrt{frac{k}{j}} quad text{for all } k < j]From ( a_i^2 = 1 ), we have ( a_i = pm 1 ). But since all ( x_i geq 0 ), and we're looking for maximum and minimum of ( sum x_i ), perhaps we can take ( a_i = 1 ) for all ( i ). But then, ( a_k a_j = 1 ), which doesn't match ( sqrt{frac{k}{j}} ).Alternatively, maybe ( a_i = sqrt{frac{c}{i}} ) for some constant ( c ). Let me test this:If ( a_i = sqrt{frac{c}{i}} ), then ( a_i^2 = frac{c}{i} ). For this to equal 1, we need ( c = i ), but ( c ) is a constant, so this doesn't work.Wait, perhaps ( a_i = sqrt{frac{i}{c}} ). Then, ( a_i^2 = frac{i}{c} ). To have ( a_i^2 = 1 ), we need ( c = i ), which again depends on ( i ), so it's not a constant.Hmm, maybe this approach isn't working. Let me think differently.Wait, another idea: perhaps the quadratic form can be expressed as ( left( sum_{i=1}^{n} sqrt{i} x_i right)^2 ). Let me check:[left( sum_{i=1}^{n} sqrt{i} x_i right)^2 = sum_{i=1}^{n} i x_i^2 + 2 sum_{1 leq k < j leq n} sqrt{kj} x_k x_j]But in the given condition, the cross terms are ( 2 sqrt{frac{k}{j}} x_k x_j ), not ( 2 sqrt{kj} x_k x_j ). So, this doesn't match.Wait, but if I factor out something from the cross terms. Let me see:Given cross term: ( 2 sqrt{frac{k}{j}} x_k x_j = 2 frac{sqrt{k}}{sqrt{j}} x_k x_j )If I let ( y_i = sqrt{i} x_i ), then ( x_i = frac{y_i}{sqrt{i}} ). Substitute into the cross term:[2 frac{sqrt{k}}{sqrt{j}} cdot frac{y_k}{sqrt{k}} cdot frac{y_j}{sqrt{j}} = 2 frac{sqrt{k}}{sqrt{j}} cdot frac{y_k}{sqrt{k}} cdot frac{y_j}{sqrt{j}} = 2 frac{y_k y_j}{j}]Wait, that's not helpful. Maybe I need a different substitution.Alternatively, perhaps I can write the quadratic form as:[sum_{i=1}^{n} x_i^2 + 2 sum_{1 leq k < j leq n} sqrt{frac{k}{j}} x_k x_j = left( sum_{i=1}^{n} x_i sqrt{frac{i}{j}} right)^2 quad text{?}]No, that doesn't seem right.Wait, perhaps I can think of the quadratic form as a sum of squares. Let me try to complete the square.Start with the first term ( x_1^2 ). Then, the cross terms involving ( x_1 ) are ( 2 sqrt{frac{1}{j}} x_1 x_j ) for ( j > 1 ). So, perhaps I can write:[x_1^2 + 2 x_1 sum_{j=2}^{n} sqrt{frac{1}{j}} x_j + sum_{j=2}^{n} x_j^2 + 2 sum_{2 leq k < j leq n} sqrt{frac{k}{j}} x_k x_j]Hmm, maybe I can group terms involving ( x_1 ) and complete the square.Let me denote ( S = sum_{j=2}^{n} sqrt{frac{1}{j}} x_j ). Then, the expression becomes:[(x_1 + S)^2 + sum_{j=2}^{n} x_j^2 + 2 sum_{2 leq k < j leq n} sqrt{frac{k}{j}} x_k x_j - 2 x_1 S - S^2]Wait, this seems messy. Maybe another approach.Wait, perhaps I can consider the quadratic form as a sum over ( j ), where each ( x_j ) is multiplied by terms involving ( x_k ) for ( k < j ).Let me write the quadratic form as:[sum_{j=1}^{n} x_j^2 + 2 sum_{j=2}^{n} sum_{k=1}^{j-1} sqrt{frac{k}{j}} x_k x_j]Now, for each ( j ), the term is ( x_j^2 + 2 sum_{k=1}^{j-1} sqrt{frac{k}{j}} x_k x_j ). Maybe I can factor this as ( left( x_j + sum_{k=1}^{j-1} sqrt{frac{k}{j}} x_k right)^2 ). Let me check:[left( x_j + sum_{k=1}^{j-1} sqrt{frac{k}{j}} x_k right)^2 = x_j^2 + 2 x_j sum_{k=1}^{j-1} sqrt{frac{k}{j}} x_k + left( sum_{k=1}^{j-1} sqrt{frac{k}{j}} x_k right)^2]But in our quadratic form, we only have ( x_j^2 + 2 sum_{k=1}^{j-1} sqrt{frac{k}{j}} x_k x_j ). So, the extra term ( left( sum_{k=1}^{j-1} sqrt{frac{k}{j}} x_k right)^2 ) is not present. Therefore, this approach doesn't directly work.Wait, but perhaps if I consider the entire quadratic form as a telescoping sum. Let me think recursively.Suppose I define ( y_j = x_j + sum_{k=1}^{j-1} sqrt{frac{k}{j}} x_k ). Then, the quadratic form would be ( sum_{j=1}^{n} y_j^2 - sum_{j=2}^{n} left( sum_{k=1}^{j-1} sqrt{frac{k}{j}} x_k right)^2 ). But this seems complicated.Alternatively, maybe I can use induction. For ( n = 1 ), it's trivial. For ( n = 2 ), we saw that the maximum is ( sqrt{4 - 2sqrt{2}} ). For ( n = 3 ), it's more involved, but perhaps the maximum is the square root of the sum of squares of differences of square roots.Wait, in the case of ( n = 2 ), the maximum was ( sqrt{4 - 2sqrt{2}} ), which can be written as ( sqrt{( sqrt{2} )^2 + ( sqrt{2} )^2 - 2 cdot sqrt{2} cdot sqrt{2} cdot frac{1}{sqrt{2}}} ). Hmm, not sure.Wait, another idea: perhaps the maximum is the sum of ( sqrt{k} - sqrt{k-1} ) for ( k = 1 ) to ( n ). Let me compute this sum:[sum_{k=1}^{n} (sqrt{k} - sqrt{k-1}) = sqrt{n} - sqrt{0} = sqrt{n}]But for ( n = 2 ), this would be ( sqrt{2} approx 1.414 ), which is larger than the maximum we found earlier, which was approximately 1.082. So, that can't be.Wait, but maybe the maximum is the square root of the sum of squares of ( sqrt{k} - sqrt{k-1} ). Let me compute that:[sqrt{ sum_{k=1}^{n} (sqrt{k} - sqrt{k-1})^2 }]For ( n = 2 ):[sqrt{ (sqrt{1} - sqrt{0})^2 + (sqrt{2} - sqrt{1})^2 } = sqrt{1 + ( sqrt{2} - 1 )^2 } = sqrt{1 + 3 - 2sqrt{2}} = sqrt{4 - 2sqrt{2}} approx 1.082]Which matches the maximum for ( n = 2 ). For ( n = 1 ):[sqrt{ (sqrt{1} - sqrt{0})^2 } = sqrt{1} = 1]Which matches. For ( n = 3 ):[sqrt{1 + ( sqrt{2} - 1 )^2 + ( sqrt{3} - sqrt{2} )^2 } = sqrt{1 + (3 - 2sqrt{2}) + (3 - 2sqrt{6} + 2)} quad text{Wait, no. Let me compute correctly:}]Wait, actually, ( (sqrt{k} - sqrt{k-1})^2 = k - 2sqrt{k(k-1)} + (k-1) = 2k - 1 - 2sqrt{k(k-1)} ). So, summing from ( k = 1 ) to ( n ):[sum_{k=1}^{n} (2k - 1 - 2sqrt{k(k-1)}) = sum_{k=1}^{n} (2k - 1) - 2 sum_{k=1}^{n} sqrt{k(k-1)}]But ( sum_{k=1}^{n} (2k - 1) = n^2 ), since it's the sum of the first ( n ) odd numbers.And ( sum_{k=1}^{n} sqrt{k(k-1)} = sum_{k=1}^{n} sqrt{k^2 - k} ). For ( k = 1 ), this is 0. For ( k = 2 ), it's ( sqrt{2} ). For ( k = 3 ), it's ( sqrt{6} ), etc. So, it's a bit messy.But for ( n = 2 ), the sum is ( 0 + sqrt{2} approx 1.414 ). So, the expression becomes:[sqrt{4 - 2 cdot 1.414} = sqrt{4 - 2.828} = sqrt{1.172} approx 1.082]Which matches. So, perhaps the maximum is indeed ( sqrt{ sum_{k=1}^{n} (sqrt{k} - sqrt{k-1})^2 } ).But let me see if this holds for ( n = 3 ). Compute:[sum_{k=1}^{3} (sqrt{k} - sqrt{k-1})^2 = (sqrt{1} - 0)^2 + (sqrt{2} - 1)^2 + (sqrt{3} - sqrt{2})^2][= 1 + (3 - 2sqrt{2}) + (3 - 2sqrt{6} + 2)]Wait, no, that's incorrect. Let me compute each term:( (sqrt{1} - 0)^2 = 1 )( (sqrt{2} - 1)^2 = 2 - 2sqrt{2} + 1 = 3 - 2sqrt{2} )( (sqrt{3} - sqrt{2})^2 = 3 - 2sqrt{6} + 2 = 5 - 2sqrt{6} )So, total sum:[1 + (3 - 2sqrt{2}) + (5 - 2sqrt{6}) = 9 - 2sqrt{2} - 2sqrt{6}]So, the square root is ( sqrt{9 - 2sqrt{2} - 2sqrt{6}} ). Let me approximate this:Compute ( 9 - 2sqrt{2} - 2sqrt{6} approx 9 - 2.828 - 4.899 approx 9 - 7.727 approx 1.273 ). So, the square root is approximately 1.128.But is this the maximum for ( n = 3 )? Let me see.Alternatively, perhaps the maximum is achieved when all variables are set such that ( x_i = sqrt{i} - sqrt{i-1} ). Wait, let me test this.Suppose ( x_i = sqrt{i} - sqrt{i-1} ) for ( i = 1, 2, ldots, n ). Then, the sum ( sum_{i=1}^{n} x_i = sqrt{n} ). But earlier, for ( n = 2 ), this would give ( sqrt{2} approx 1.414 ), which is higher than the maximum we found, which was approximately 1.082. So, that can't be.Wait, but perhaps the maximum is achieved when ( x_i ) are proportional to ( sqrt{i} - sqrt{i-1} ). Let me think.If I set ( x_i = c (sqrt{i} - sqrt{i-1}) ), then the sum ( sum x_i = c sqrt{n} ). Now, substitute into the quadratic form:[sum_{i=1}^{n} x_i^2 + 2 sum_{1 leq k < j leq n} sqrt{frac{k}{j}} x_k x_j = c^2 left( sum_{i=1}^{n} (sqrt{i} - sqrt{i-1})^2 + 2 sum_{1 leq k < j leq n} sqrt{frac{k}{j}} (sqrt{k} - sqrt{k-1})(sqrt{j} - sqrt{j-1}) right)]This seems complicated, but perhaps it simplifies. Let me compute the first sum:[sum_{i=1}^{n} (sqrt{i} - sqrt{i-1})^2 = sum_{i=1}^{n} (i - 2sqrt{i(i-1)} + (i-1)) = sum_{i=1}^{n} (2i - 1 - 2sqrt{i(i-1)})]As before, this is ( n^2 - 2 sum_{i=1}^{n} sqrt{i(i-1)} ).Now, the cross terms:[2 sum_{1 leq k < j leq n} sqrt{frac{k}{j}} (sqrt{k} - sqrt{k-1})(sqrt{j} - sqrt{j-1})]Let me factor out the constants:[2 sum_{1 leq k < j leq n} sqrt{frac{k}{j}} (sqrt{k} - sqrt{k-1})(sqrt{j} - sqrt{j-1}) = 2 sum_{1 leq k < j leq n} left( frac{sqrt{k}}{sqrt{j}} (sqrt{k} - sqrt{k-1})(sqrt{j} - sqrt{j-1}) right)]Simplify inside:[frac{sqrt{k}}{sqrt{j}} (sqrt{k} - sqrt{k-1})(sqrt{j} - sqrt{j-1}) = frac{k - sqrt{k(k-1)}}{sqrt{j}} (sqrt{j} - sqrt{j-1})]This is getting too messy. Maybe I should consider a different approach.Wait, going back to the Lagrangian method, perhaps I can assume that all variables are positive and find a relationship between them.From the partial derivatives, for each ( i ):[1 = 2 lambda x_i + 2 lambda sum_{j=1}^{i-1} sqrt{frac{j}{i}} x_j + 2 lambda sum_{j=i+1}^{n} sqrt{frac{i}{j}} x_j]Let me denote ( S_i = sum_{j=1}^{i-1} sqrt{frac{j}{i}} x_j + sum_{j=i+1}^{n} sqrt{frac{i}{j}} x_j ). Then, the equation becomes:[1 = 2 lambda (x_i + S_i)]So, ( x_i + S_i = frac{1}{2 lambda} ) for all ( i ).This suggests that ( x_i + S_i ) is constant for all ( i ). Let me denote this constant as ( C = frac{1}{2 lambda} ).So, for each ( i ):[x_i + sum_{j=1}^{i-1} sqrt{frac{j}{i}} x_j + sum_{j=i+1}^{n} sqrt{frac{i}{j}} x_j = C]This is a system of equations that relates each ( x_i ) to the others.Let me try to write this for ( i = 1 ):For ( i = 1 ), the equation is:[x_1 + sum_{j=2}^{n} sqrt{frac{1}{j}} x_j = C]Similarly, for ( i = 2 ):[x_2 + sqrt{frac{1}{2}} x_1 + sum_{j=3}^{n} sqrt{frac{2}{j}} x_j = C]For ( i = 3 ):[x_3 + sqrt{frac{1}{3}} x_1 + sqrt{frac{2}{3}} x_2 + sum_{j=4}^{n} sqrt{frac{3}{j}} x_j = C]And so on, until ( i = n ):[x_n + sum_{j=1}^{n-1} sqrt{frac{j}{n}} x_j = C]This system seems quite involved, but perhaps there's a pattern or a way to express ( x_i ) in terms of ( x_{i+1} ) or something similar.Alternatively, maybe I can consider the ratios between consecutive ( x_i ). Let me assume that ( x_{i+1} = r x_i ) for some ratio ( r ). Let me see if this assumption can satisfy the equations.Assume ( x_{i+1} = r x_i ). Then, ( x_2 = r x_1 ), ( x_3 = r x_2 = r^2 x_1 ), and so on, up to ( x_n = r^{n-1} x_1 ).Now, substitute into the equation for ( i = 1 ):[x_1 + sum_{j=2}^{n} sqrt{frac{1}{j}} x_j = x_1 + sum_{j=2}^{n} sqrt{frac{1}{j}} r^{j-1} x_1 = C]Factor out ( x_1 ):[x_1 left( 1 + sum_{j=2}^{n} sqrt{frac{1}{j}} r^{j-1} right) = C]Similarly, for ( i = 2 ):[x_2 + sqrt{frac{1}{2}} x_1 + sum_{j=3}^{n} sqrt{frac{2}{j}} x_j = r x_1 + sqrt{frac{1}{2}} x_1 + sum_{j=3}^{n} sqrt{frac{2}{j}} r^{j-1} x_1 = C]Factor out ( x_1 ):[x_1 left( r + sqrt{frac{1}{2}} + sum_{j=3}^{n} sqrt{frac{2}{j}} r^{j-1} right) = C]Since both expressions equal ( C ), we can set them equal:[1 + sum_{j=2}^{n} sqrt{frac{1}{j}} r^{j-1} = r + sqrt{frac{1}{2}} + sum_{j=3}^{n} sqrt{frac{2}{j}} r^{j-1}]Simplify:[1 + sqrt{frac{1}{2}} r + sum_{j=3}^{n} sqrt{frac{1}{j}} r^{j-1} = r + sqrt{frac{1}{2}} + sum_{j=3}^{n} sqrt{frac{2}{j}} r^{j-1}]Subtract ( r + sqrt{frac{1}{2}} ) from both sides:[1 - r - sqrt{frac{1}{2}} + sqrt{frac{1}{2}} r + sum_{j=3}^{n} sqrt{frac{1}{j}} r^{j-1} - sum_{j=3}^{n} sqrt{frac{2}{j}} r^{j-1} = 0]Factor the sums:[1 - r - sqrt{frac{1}{2}} + sqrt{frac{1}{2}} r + sum_{j=3}^{n} left( sqrt{frac{1}{j}} - sqrt{frac{2}{j}} right) r^{j-1} = 0]This equation must hold for all ( j geq 3 ), which seems difficult unless the coefficients of each power of ( r ) are zero. But this is a complicated equation, and it's not clear if such an ( r ) exists that satisfies it for all ( j ).Therefore, perhaps the assumption ( x_{i+1} = r x_i ) is not valid. Maybe a different approach is needed.Wait, going back to the earlier idea, if the maximum is ( sqrt{ sum_{k=1}^{n} (sqrt{k} - sqrt{k-1})^2 } ), then perhaps this is the general case.Let me test this for ( n = 3 ). Compute:[sum_{k=1}^{3} (sqrt{k} - sqrt{k-1})^2 = 1 + ( sqrt{2} - 1 )^2 + ( sqrt{3} - sqrt{2} )^2 approx 1 + (1.414 - 1)^2 + (1.732 - 1.414)^2 approx 1 + 0.171 + 0.101 approx 1.272]So, the square root is approximately 1.128.Now, let me see if this makes sense. If I set ( x_i = sqrt{k} - sqrt{k-1} ), then the sum ( sum x_i = sqrt{3} approx 1.732 ), which is larger than 1.128, so that can't be.Wait, but earlier for ( n = 2 ), the maximum was ( sqrt{4 - 2sqrt{2}} approx 1.082 ), which is less than ( sqrt{2} approx 1.414 ). So, perhaps the maximum is indeed ( sqrt{ sum_{k=1}^{n} (sqrt{k} - sqrt{k-1})^2 } ).But how does this relate to the quadratic form?Wait, perhaps the quadratic form can be expressed as ( left( sum_{k=1}^{n} (sqrt{k} - sqrt{k-1}) x_k right)^2 ). Let me check:[left( sum_{k=1}^{n} (sqrt{k} - sqrt{k-1}) x_k right)^2 = sum_{k=1}^{n} (sqrt{k} - sqrt{k-1})^2 x_k^2 + 2 sum_{1 leq k < j leq n} (sqrt{k} - sqrt{k-1})(sqrt{j} - sqrt{j-1}) x_k x_j]But in the given quadratic form, the cross terms are ( 2 sqrt{frac{k}{j}} x_k x_j ). So, unless ( (sqrt{k} - sqrt{k-1})(sqrt{j} - sqrt{j-1}) = sqrt{frac{k}{j}} ), which is not generally true, this approach doesn't work.Wait, let me compute ( (sqrt{k} - sqrt{k-1})(sqrt{j} - sqrt{j-1}) ) for ( k < j ):[(sqrt{k} - sqrt{k-1})(sqrt{j} - sqrt{j-1}) = sqrt{k j} - sqrt{k(j-1)} - sqrt{(k-1)j} + sqrt{(k-1)(j-1)}]This is much more complicated than ( sqrt{frac{k}{j}} ). So, this approach doesn't seem to help.Hmm, maybe I need to think differently. Let me consider the quadratic form as a positive definite matrix and use the fact that the maximum of ( mathbf{1}^T mathbf{x} ) subject to ( mathbf{x}^T A mathbf{x} = 1 ) is ( sqrt{ mathbf{1}^T A^{-1} mathbf{1} } ).So, if I can compute ( A^{-1} ), then ( mathbf{1}^T A^{-1} mathbf{1} ) would give me the square of the maximum value.But computing ( A^{-1} ) for a general ( n ) is non-trivial. However, perhaps there's a pattern or a way to express ( A^{-1} ) in terms of the given structure.Alternatively, perhaps the quadratic form can be expressed as ( left( sum_{i=1}^{n} sqrt{i} x_i right)^2 ), but earlier that didn't match because the cross terms were different.Wait, another idea: perhaps the quadratic form is equal to ( left( sum_{i=1}^{n} x_i sqrt{frac{i}{n}} right)^2 ). Let me test this for ( n = 2 ):[left( x_1 sqrt{frac{1}{2}} + x_2 sqrt{frac{2}{2}} right)^2 = frac{1}{2} x_1^2 + x_2^2 + 2 cdot sqrt{frac{1}{2}} x_1 x_2]Which matches the given quadratic form for ( n = 2 ). So, for ( n = 2 ), the quadratic form is ( left( frac{x_1}{sqrt{2}} + x_2 right)^2 ).Similarly, for ( n = 3 ), perhaps the quadratic form is ( left( frac{x_1}{sqrt{3}} + frac{x_2}{sqrt{3}} + x_3 right)^2 ). Let me check:[left( frac{x_1}{sqrt{3}} + frac{x_2}{sqrt{3}} + x_3 right)^2 = frac{x_1^2}{3} + frac{x_2^2}{3} + x_3^2 + 2 cdot frac{x_1 x_2}{3} + 2 cdot frac{x_1 x_3}{sqrt{3}} + 2 cdot frac{x_2 x_3}{sqrt{3}}]But the given quadratic form for ( n = 3 ) is:[x_1^2 + x_2^2 + x_3^2 + 2 sqrt{frac{1}{2}} x_1 x_2 + 2 sqrt{frac{1}{3}} x_1 x_3 + 2 sqrt{frac{2}{3}} x_2 x_3]These don't match because the coefficients are different. So, this approach doesn't work for ( n = 3 ).Wait, but for ( n = 2 ), the quadratic form can be expressed as ( left( frac{x_1}{sqrt{2}} + x_2 right)^2 ). Maybe for general ( n ), the quadratic form can be expressed as ( left( sum_{i=1}^{n} sqrt{frac{i}{n}} x_i right)^2 ). Let me test for ( n = 3 ):[left( frac{x_1}{sqrt{3}} + frac{x_2}{sqrt{3}} + frac{x_3}{sqrt{3}} right)^2 = frac{x_1^2}{3} + frac{x_2^2}{3} + frac{x_3^2}{3} + 2 cdot frac{x_1 x_2}{3} + 2 cdot frac{x_1 x_3}{3} + 2 cdot frac{x_2 x_3}{3}]But the given quadratic form for ( n = 3 ) has different coefficients, so this doesn't match.Hmm, perhaps the quadratic form can be expressed as ( left( sum_{i=1}^{n} sqrt{frac{i}{n}} x_i right)^2 ) only for specific ( n ), not in general.Wait, another idea: perhaps the quadratic form is a perfect square of a linear combination where the coefficients are ( sqrt{frac{k}{j}} ) for ( k < j ). But I don't see a straightforward way to express it.Alternatively, perhaps I can use the Cauchy-Schwarz inequality on the given quadratic form. Let me consider the sum ( sum x_i ) and relate it to the quadratic form.By Cauchy-Schwarz,[left( sum_{i=1}^{n} x_i right)^2 leq left( sum_{i=1}^{n} a_i^2 right) left( sum_{i=1}^{n} left( frac{x_i}{a_i} right)^2 right)]But I'm not sure how to choose ( a_i ) to make this work with the given quadratic form.Wait, another approach: since the quadratic form is given, perhaps I can use the fact that it's equal to 1, and then express the sum ( sum x_i ) in terms of it.Let me denote ( S = sum x_i ). Then, the quadratic form is:[sum x_i^2 + 2 sum_{k < j} sqrt{frac{k}{j}} x_k x_j = 1]Note that ( S^2 = sum x_i^2 + 2 sum_{k < j} x_k x_j ). So, the given quadratic form is:[S^2 - 2 sum_{k < j} left( 1 - sqrt{frac{k}{j}} right) x_k x_j = 1]Thus,[S^2 = 1 + 2 sum_{k < j} left( 1 - sqrt{frac{k}{j}} right) x_k x_j]Since all ( x_i geq 0 ), the term ( sum_{k < j} left( 1 - sqrt{frac{k}{j}} right) x_k x_j geq 0 ), because ( 1 - sqrt{frac{k}{j}} geq 0 ) for ( k < j ).Therefore, ( S^2 geq 1 ), which implies ( S geq 1 ). So, the minimum value of ( S ) is 1, achieved when all cross terms are zero, i.e., when only one ( x_i ) is non-zero (equal to 1) and the rest are zero.Now, for the maximum, since ( S^2 = 1 + 2 sum_{k < j} left( 1 - sqrt{frac{k}{j}} right) x_k x_j ), and the cross terms are non-negative, ( S^2 ) can be larger than 1. To find the maximum ( S ), we need to maximize ( S ) subject to the given quadratic form being 1.This is similar to finding the maximum of a linear function subject to a quadratic constraint, which can be done using the method of Lagrange multipliers or by recognizing it as the generalized Rayleigh quotient.In such cases, the maximum value is given by the square root of the largest eigenvalue of the matrix ( A^{-1} ) multiplied by the vector ( mathbf{1} ). However, computing this for a general ( n ) is non-trivial.But earlier, for ( n = 2 ), we saw that the maximum was ( sqrt{4 - 2sqrt{2}} ), which can be written as ( sqrt{ sum_{k=1}^{2} (sqrt{k} - sqrt{k-1})^2 } ). For ( n = 3 ), it would be ( sqrt{ sum_{k=1}^{3} (sqrt{k} - sqrt{k-1})^2 } ), and so on.Thus, it seems that the maximum value of ( S ) is ( sqrt{ sum_{k=1}^{n} (sqrt{k} - sqrt{k-1})^2 } ).Let me compute this sum:[sum_{k=1}^{n} (sqrt{k} - sqrt{k-1})^2 = sum_{k=1}^{n} (k - 2sqrt{k(k-1)} + (k-1)) = sum_{k=1}^{n} (2k - 1 - 2sqrt{k(k-1)})]This simplifies to:[2 sum_{k=1}^{n} k - sum_{k=1}^{n} 1 - 2 sum_{k=1}^{n} sqrt{k(k-1)}][= 2 cdot frac{n(n+1)}{2} - n - 2 sum_{k=1}^{n} sqrt{k(k-1)}][= n(n+1) - n - 2 sum_{k=1}^{n} sqrt{k(k-1)}][= n^2 - 2 sum_{k=1}^{n} sqrt{k(k-1)}]But this doesn't seem to simplify easily. However, for the purpose of expressing the maximum value, we can leave it as ( sqrt{ sum_{k=1}^{n} (sqrt{k} - sqrt{k-1})^2 } ).Therefore, the maximum value of ( sum x_i ) is ( sqrt{ sum_{k=1}^{n} (sqrt{k} - sqrt{k-1})^2 } ), and the minimum value is 1.So, to summarize:- The minimum value of ( sum x_i ) is 1, achieved when one ( x_i = 1 ) and the rest are 0.- The maximum value of ( sum x_i ) is ( sqrt{ sum_{k=1}^{n} (sqrt{k} - sqrt{k-1})^2 } ).But let me compute this sum more explicitly. Notice that:[sum_{k=1}^{n} (sqrt{k} - sqrt{k-1})^2 = sum_{k=1}^{n} (k - 2sqrt{k(k-1)} + (k-1)) = sum_{k=1}^{n} (2k - 1 - 2sqrt{k(k-1)})]But this is equal to:[2 sum_{k=1}^{n} k - sum_{k=1}^{n} 1 - 2 sum_{k=1}^{n} sqrt{k(k-1)}][= 2 cdot frac{n(n+1)}{2} - n - 2 sum_{k=1}^{n} sqrt{k(k-1)}][= n(n+1) - n - 2 sum_{k=1}^{n} sqrt{k(k-1)}][= n^2 - 2 sum_{k=1}^{n} sqrt{k(k-1)}]But ( sum_{k=1}^{n} sqrt{k(k-1)} ) is a bit complicated. However, for the purpose of expressing the maximum value, we can leave it as is.Alternatively, perhaps there's a telescoping nature when considering the square root terms. Let me see:Note that ( sqrt{k(k-1)} = sqrt{k^2 - k} ), which doesn't telescope directly. However, perhaps integrating or approximating, but that's beyond the scope here.Therefore, the maximum value is ( sqrt{ sum_{k=1}^{n} (sqrt{k} - sqrt{k-1})^2 } ), and the minimum is 1.So, final answer:Minimum value: ( boxed{1} )Maximum value: ( boxed{ sqrt{ sum_{k=1}^{n} (sqrt{k} - sqrt{k-1})^2 } } )</think>
❓:The vertices of a cube ABCD-A1B1C1D1 with edge length a are all on the surface of a sphere O. E and F are the midpoints of edges AA1 and DD1, respectively. The length of the line segment cut off by sphere O from line EF is __________.
🧠:<think>Alright, so I'm trying to solve this geometry problem about a cube inscribed in a sphere. The cube has edge length ( a ), and all its vertices lie on the surface of the sphere ( O ). Points ( E ) and ( F ) are the midpoints of edges ( AA_1 ) and ( DD_1 ), respectively. I need to find the length of the line segment cut off by the sphere ( O ) from the line ( EF ).First, let me visualize the cube and the sphere. Since all the vertices of the cube are on the sphere, the sphere must be the circumscribed sphere (circum sphere) of the cube. That means the center of the sphere coincides with the center of the cube.Now, to find the radius of the sphere, I remember that for a cube with edge length ( a ), the space diagonal (the diagonal connecting two opposite vertices through the interior of the cube) is ( asqrt{3} ). Since the sphere is circumscribed around the cube, the radius of the sphere is half of this space diagonal. So, the radius ( R ) is:[R = frac{asqrt{3}}{2}]Okay, that makes sense. Now, I need to figure out where points ( E ) and ( F ) are located. ( E ) is the midpoint of edge ( AA_1 ), and ( F ) is the midpoint of edge ( DD_1 ). Let me consider the cube in a coordinate system to make things clearer.Let's place the cube such that its center is at the origin ( (0, 0, 0) ). Then, the vertices of the cube can be represented with coordinates where each coordinate is either ( frac{a}{2} ) or ( -frac{a}{2} ). For example, vertex ( A ) could be at ( left( frac{a}{2}, frac{a}{2}, frac{a}{2} right) ), and vertex ( A_1 ) would be at ( left( frac{a}{2}, frac{a}{2}, -frac{a}{2} right) ). Similarly, vertex ( D ) would be at ( left( -frac{a}{2}, frac{a}{2}, frac{a}{2} right) ), and vertex ( D_1 ) at ( left( -frac{a}{2}, frac{a}{2}, -frac{a}{2} right) ).Given that, point ( E ) is the midpoint of ( AA_1 ), so its coordinates would be the average of the coordinates of ( A ) and ( A_1 ):[E = left( frac{frac{a}{2} + frac{a}{2}}{2}, frac{frac{a}{2} + frac{a}{2}}{2}, frac{frac{a}{2} + (-frac{a}{2})}{2} right) = left( frac{a}{2}, frac{a}{2}, 0 right)]Similarly, point ( F ) is the midpoint of ( DD_1 ), so its coordinates are:[F = left( frac{-frac{a}{2} + (-frac{a}{2})}{2}, frac{frac{a}{2} + frac{a}{2}}{2}, frac{frac{a}{2} + (-frac{a}{2})}{2} right) = left( -frac{a}{2}, frac{a}{2}, 0 right)]Wait, hold on. That doesn't seem right. If I take the midpoint of ( DD_1 ), which goes from ( D ) at ( left( -frac{a}{2}, frac{a}{2}, frac{a}{2} right) ) to ( D_1 ) at ( left( -frac{a}{2}, frac{a}{2}, -frac{a}{2} right) ), the midpoint should be:[F = left( -frac{a}{2}, frac{a}{2}, 0 right)]Yes, that's correct. So, both ( E ) and ( F ) lie on the plane ( z = 0 ), which is the center plane of the cube.Now, the line ( EF ) connects points ( E ) and ( F ). Let me write the parametric equations for line ( EF ). Since ( E ) is ( left( frac{a}{2}, frac{a}{2}, 0 right) ) and ( F ) is ( left( -frac{a}{2}, frac{a}{2}, 0 right) ), the direction vector from ( E ) to ( F ) is ( left( -a, 0, 0 right) ). So, the parametric equations can be written as:[x = frac{a}{2} - a t][y = frac{a}{2}][z = 0]where ( t ) ranges from 0 to 1.But I need to find where this line intersects the sphere. The sphere has its center at the origin and radius ( R = frac{asqrt{3}}{2} ). The equation of the sphere is:[x^2 + y^2 + z^2 = R^2 = left( frac{asqrt{3}}{2} right)^2 = frac{3a^2}{4}]Now, substitute the parametric equations of line ( EF ) into the sphere's equation:[left( frac{a}{2} - a t right)^2 + left( frac{a}{2} right)^2 + 0^2 = frac{3a^2}{4}]Let me expand and simplify this equation:First, expand ( left( frac{a}{2} - a t right)^2 ):[left( frac{a}{2} - a t right)^2 = left( frac{a}{2} right)^2 - 2 cdot frac{a}{2} cdot a t + (a t)^2 = frac{a^2}{4} - a^2 t + a^2 t^2]So, substituting back into the sphere equation:[left( frac{a^2}{4} - a^2 t + a^2 t^2 right) + frac{a^2}{4} = frac{3a^2}{4}]Combine like terms:[frac{a^2}{4} + frac{a^2}{4} - a^2 t + a^2 t^2 = frac{3a^2}{4}][frac{a^2}{2} - a^2 t + a^2 t^2 = frac{3a^2}{4}]Subtract ( frac{3a^2}{4} ) from both sides:[frac{a^2}{2} - a^2 t + a^2 t^2 - frac{3a^2}{4} = 0][left( frac{a^2}{2} - frac{3a^2}{4} right) - a^2 t + a^2 t^2 = 0][- frac{a^2}{4} - a^2 t + a^2 t^2 = 0]Multiply both sides by ( -1 ) to make it easier:[frac{a^2}{4} + a^2 t - a^2 t^2 = 0]Factor out ( a^2 ):[a^2 left( frac{1}{4} + t - t^2 right) = 0]Since ( a ) is not zero, we can divide both sides by ( a^2 ):[frac{1}{4} + t - t^2 = 0]Let me rewrite this quadratic equation:[- t^2 + t + frac{1}{4} = 0][t^2 - t - frac{1}{4} = 0]Now, solving for ( t ) using the quadratic formula:[t = frac{1 pm sqrt{1 + 1}}{2} = frac{1 pm sqrt{2}}{2}]So, the two solutions are:[t = frac{1 + sqrt{2}}{2} quad text{and} quad t = frac{1 - sqrt{2}}{2}]But wait, in our parametric equation, ( t ) ranges from 0 to 1 because it's moving from ( E ) to ( F ). Let me compute the numerical values:[frac{1 + sqrt{2}}{2} approx frac{1 + 1.4142}{2} approx frac{2.4142}{2} approx 1.2071][frac{1 - sqrt{2}}{2} approx frac{1 - 1.4142}{2} approx frac{-0.4142}{2} approx -0.2071]So, both solutions are outside the range ( [0, 1] ). That means the line ( EF ) doesn't intersect the sphere within the segment ( EF ). Hmm, that can't be right because the sphere is circumscribed around the cube, and ( EF ) is inside the cube, so it should intersect the sphere at two points.Wait, maybe I made a mistake in setting up the parametric equations or in the substitution.Let me double-check the parametric equations. I had:[x = frac{a}{2} - a t][y = frac{a}{2}][z = 0]But actually, since ( E ) is at ( left( frac{a}{2}, frac{a}{2}, 0 right) ) and ( F ) is at ( left( -frac{a}{2}, frac{a}{2}, 0 right) ), the direction vector is ( (-a, 0, 0) ), so the parametric equations are correct.Wait, but if the sphere is circumscribed around the cube, then all the vertices are on the sphere, but points ( E ) and ( F ) are midpoints, so they are inside the sphere. Therefore, the line ( EF ) is entirely inside the sphere, right? Because both ( E ) and ( F ) are inside the sphere, so the entire segment ( EF ) is inside the sphere.But the problem says "the length of the line segment cut off by sphere ( O ) from line ( EF )." If the entire segment ( EF ) is inside the sphere, then the sphere doesn't cut off any part of ( EF ); the entire segment remains inside. But that contradicts the problem statement, which implies that part of ( EF ) is cut off by the sphere.Wait, perhaps I made a mistake in the coordinates of ( E ) and ( F ). Let me re-examine that.I assumed the cube is centered at the origin, so each vertex has coordinates ( pm frac{a}{2} ) in each axis. So, vertex ( A ) is ( left( frac{a}{2}, frac{a}{2}, frac{a}{2} right) ), and ( A_1 ) is ( left( frac{a}{2}, frac{a}{2}, -frac{a}{2} right) ). Therefore, midpoint ( E ) is indeed ( left( frac{a}{2}, frac{a}{2}, 0 right) ). Similarly, vertex ( D ) is ( left( -frac{a}{2}, frac{a}{2}, frac{a}{2} right) ), and ( D_1 ) is ( left( -frac{a}{2}, frac{a}{2}, -frac{a}{2} right) ), so midpoint ( F ) is ( left( -frac{a}{2}, frac{a}{2}, 0 right) ). So, my coordinates for ( E ) and ( F ) seem correct.Wait, but if both ( E ) and ( F ) are on the sphere, then the segment ( EF ) would lie on the sphere's surface. But no, because the distance from the center to ( E ) is:[sqrt{left( frac{a}{2} right)^2 + left( frac{a}{2} right)^2 + 0^2} = sqrt{frac{a^2}{4} + frac{a^2}{4}} = sqrt{frac{a^2}{2}} = frac{a}{sqrt{2}} approx 0.707a]But the radius of the sphere is ( frac{asqrt{3}}{2} approx 0.866a ), which is larger. So, ( E ) and ( F ) are inside the sphere, meaning the line ( EF ) passes through the sphere, entering at one point and exiting at another. Therefore, the segment of ( EF ) that lies inside the sphere is between these two intersection points.Wait, but earlier when I solved the equation, the ( t ) values were outside the range ( [0, 1] ), implying that the intersections are outside the segment ( EF ). That seems contradictory.Wait, perhaps I made a mistake in the parametrization. Let me think differently. Maybe I should parametrize the line ( EF ) differently, not just from ( E ) to ( F ), but as an infinite line.Let me consider the line ( EF ) as an infinite line, not just the segment between ( E ) and ( F ). So, the parametric equations can be written as:[x = frac{a}{2} - a t][y = frac{a}{2}][z = 0]where ( t ) can be any real number.Then, substituting into the sphere equation:[left( frac{a}{2} - a t right)^2 + left( frac{a}{2} right)^2 + 0^2 = frac{3a^2}{4}]Which simplifies to:[left( frac{a}{2} - a t right)^2 + frac{a^2}{4} = frac{3a^2}{4}]Expanding ( left( frac{a}{2} - a t right)^2 ):[frac{a^2}{4} - a^2 t + a^2 t^2]So, substituting back:[frac{a^2}{4} - a^2 t + a^2 t^2 + frac{a^2}{4} = frac{3a^2}{4}][frac{a^2}{2} - a^2 t + a^2 t^2 = frac{3a^2}{4}]Subtract ( frac{3a^2}{4} ):[frac{a^2}{2} - a^2 t + a^2 t^2 - frac{3a^2}{4} = 0][- frac{a^2}{4} - a^2 t + a^2 t^2 = 0]Divide by ( a^2 ):[- frac{1}{4} - t + t^2 = 0][t^2 - t - frac{1}{4} = 0]Solving this quadratic equation:[t = frac{1 pm sqrt{1 + 1}}{2} = frac{1 pm sqrt{2}}{2}]So, ( t = frac{1 + sqrt{2}}{2} ) and ( t = frac{1 - sqrt{2}}{2} ).Now, these are the parameter values where the line ( EF ) intersects the sphere. Since ( t ) can be any real number, these are the two points where the line enters and exits the sphere.But the segment ( EF ) is only from ( t = 0 ) to ( t = 1 ). So, we need to see if these intersection points lie within this range.Compute ( frac{1 + sqrt{2}}{2} approx frac{1 + 1.4142}{2} approx 1.2071 ), which is greater than 1.Compute ( frac{1 - sqrt{2}}{2} approx frac{1 - 1.4142}{2} approx -0.2071 ), which is less than 0.So, both intersection points are outside the segment ( EF ). That means the entire segment ( EF ) lies inside the sphere, and the sphere doesn't cut off any part of ( EF ). But that contradicts the problem statement, which asks for the length of the segment cut off by the sphere from line ( EF ).Wait, maybe I misinterpreted the problem. It says "the line segment cut off by sphere ( O ) from line ( EF )." So, it's not necessarily the segment between ( E ) and ( F ), but the segment of line ( EF ) that lies inside the sphere.But if the entire segment ( EF ) is inside the sphere, then the length cut off is just the length of ( EF ). Let me compute the length of ( EF ).Since ( E ) is ( left( frac{a}{2}, frac{a}{2}, 0 right) ) and ( F ) is ( left( -frac{a}{2}, frac{a}{2}, 0 right) ), the distance between them is:[sqrt{left( frac{a}{2} - (-frac{a}{2}) right)^2 + left( frac{a}{2} - frac{a}{2} right)^2 + (0 - 0)^2} = sqrt{left( a right)^2 + 0 + 0} = a]So, the length of ( EF ) is ( a ). But earlier, I thought the sphere cuts off a segment of length ( asqrt{3} ), which is longer than ( a ). That doesn't make sense because the sphere's diameter is ( asqrt{3} ), which is longer than ( a ).Wait, perhaps I made a mistake in assuming the line ( EF ) is along the space diagonal. Let me think again.Actually, in the cube, the space diagonal goes from a vertex to the opposite vertex, passing through the center. But in this case, ( EF ) is not a space diagonal. It's a line connecting midpoints of two edges that are not opposite.Wait, no, actually, in the cube, ( EF ) connects midpoints of edges ( AA_1 ) and ( DD_1 ). Let me visualize this.In the cube, edge ( AA_1 ) is a vertical edge from ( A ) to ( A_1 ), and edge ( DD_1 ) is another vertical edge from ( D ) to ( D_1 ). The midpoints ( E ) and ( F ) are halfway up these edges.So, in the coordinate system, ( E ) is ( left( frac{a}{2}, frac{a}{2}, 0 right) ) and ( F ) is ( left( -frac{a}{2}, frac{a}{2}, 0 right) ). So, the line ( EF ) lies in the plane ( y = frac{a}{2} ) and ( z = 0 ), which is a horizontal plane cutting through the cube.Wait, but in this case, the line ( EF ) is along the x-axis in this plane, from ( frac{a}{2} ) to ( -frac{a}{2} ). So, it's a straight line of length ( a ) inside the sphere.But earlier, when I solved for the intersections, the line ( EF ) (as an infinite line) intersects the sphere at ( t = frac{1 pm sqrt{2}}{2} ), which are outside the segment ( EF ). Therefore, the entire segment ( EF ) lies inside the sphere, and the sphere doesn't cut off any part of it. So, the length cut off by the sphere from line ( EF ) is the entire length of ( EF ), which is ( a ).But that contradicts my initial thought that the length is ( asqrt{3} ). Wait, maybe I made a mistake in the initial assumption that ( EF ) is along the space diagonal. It's not; it's along a face diagonal in the plane ( y = frac{a}{2} ), ( z = 0 ).Wait, let me compute the distance from the center of the sphere to the line ( EF ). If this distance is less than the radius, then the line intersects the sphere at two points. If it's equal, it's tangent, and if it's greater, it doesn't intersect.The distance from the center (origin) to the line ( EF ) can be computed using the formula for the distance from a point to a line in 3D.Given a line defined by two points ( E ) and ( F ), the distance ( d ) from the origin to this line is:[d = frac{|vec{OE} times vec{OF}|}{|vec{EF}|}]Wait, no, the formula is:[d = frac{|vec{OE} times vec{OF}|}{|vec{EF}|}]But actually, the correct formula is:Given a line through points ( P ) and ( Q ), the distance from a point ( O ) to the line is:[d = frac{|vec{OP} times vec{OQ}|}{|vec{PQ}|}]Wait, no, that's not quite right. The correct formula is:[d = frac{|vec{OP} times vec{v}|}{|vec{v}|}]where ( vec{v} ) is the direction vector of the line.In our case, the line ( EF ) has direction vector ( vec{EF} = F - E = left( -a, 0, 0 right) ). The vector from the origin to point ( E ) is ( vec{OE} = left( frac{a}{2}, frac{a}{2}, 0 right) ).So, the cross product ( vec{OE} times vec{EF} ) is:[vec{OE} times vec{EF} = begin{vmatrix}mathbf{i} & mathbf{j} & mathbf{k} frac{a}{2} & frac{a}{2} & 0 - a & 0 & 0 end{vmatrix}= mathbf{i} left( frac{a}{2} cdot 0 - 0 cdot 0 right) - mathbf{j} left( frac{a}{2} cdot 0 - (-a) cdot 0 right) + mathbf{k} left( frac{a}{2} cdot 0 - (-a) cdot frac{a}{2} right)][= 0mathbf{i} - 0mathbf{j} + left( 0 - (-frac{a^2}{2}) right)mathbf{k} = frac{a^2}{2}mathbf{k}]So, the magnitude of this cross product is ( frac{a^2}{2} ).The magnitude of the direction vector ( vec{EF} ) is ( sqrt{(-a)^2 + 0^2 + 0^2} = a ).Therefore, the distance ( d ) from the origin to the line ( EF ) is:[d = frac{frac{a^2}{2}}{a} = frac{a}{2}]So, the distance from the center of the sphere to the line ( EF ) is ( frac{a}{2} ), which is less than the radius ( frac{asqrt{3}}{2} ). Therefore, the line ( EF ) intersects the sphere at two points.Wait, but earlier when I solved for ( t ), the intersection points were at ( t approx 1.2071 ) and ( t approx -0.2071 ), which are outside the segment ( EF ) (which is from ( t = 0 ) to ( t = 1 )). So, the sphere intersects the infinite line ( EF ) at two points outside the segment ( EF ). Therefore, the entire segment ( EF ) lies inside the sphere, and the sphere doesn't cut off any part of ( EF ). So, the length cut off by the sphere from line ( EF ) is the entire length of ( EF ), which is ( a ).But the problem asks for the length of the line segment cut off by the sphere from line ( EF ). If the entire segment ( EF ) is inside the sphere, then the cut-off segment is the entire ( EF ), so the length is ( a ).However, this contradicts the initial thought that the length is ( asqrt{3} ). Maybe I need to reconsider.Wait, perhaps I made a mistake in the parametrization or in the interpretation of the line ( EF ). Let me think differently.Alternatively, maybe the line ( EF ) is not the segment between ( E ) and ( F ), but the entire line extending beyond ( E ) and ( F ). In that case, the sphere would intersect this line at two points, and the segment between these two intersection points is the part of the line inside the sphere. The length of this segment would be the length cut off by the sphere.Given that, let's compute the distance between the two intersection points.From earlier, the parameter ( t ) at the intersection points is ( t = frac{1 pm sqrt{2}}{2} ).So, the two points are:For ( t = frac{1 + sqrt{2}}{2} ):[x = frac{a}{2} - a cdot frac{1 + sqrt{2}}{2} = frac{a}{2} - frac{a}{2} - frac{asqrt{2}}{2} = -frac{asqrt{2}}{2}][y = frac{a}{2}][z = 0]So, point ( P ) is ( left( -frac{asqrt{2}}{2}, frac{a}{2}, 0 right) ).For ( t = frac{1 - sqrt{2}}{2} ):[x = frac{a}{2} - a cdot frac{1 - sqrt{2}}{2} = frac{a}{2} - frac{a}{2} + frac{asqrt{2}}{2} = frac{asqrt{2}}{2}][y = frac{a}{2}][z = 0]So, point ( Q ) is ( left( frac{asqrt{2}}{2}, frac{a}{2}, 0 right) ).Now, the distance between points ( P ) and ( Q ) is:[sqrt{left( frac{asqrt{2}}{2} - (-frac{asqrt{2}}{2}) right)^2 + left( frac{a}{2} - frac{a}{2} right)^2 + (0 - 0)^2} = sqrt{left( asqrt{2} right)^2 + 0 + 0} = sqrt{2a^2} = asqrt{2}]So, the length of the segment cut off by the sphere from line ( EF ) is ( asqrt{2} ).But wait, earlier I thought the length was ( asqrt{3} ), but that was based on a misunderstanding of the line's position. Now, with this calculation, it's ( asqrt{2} ).But let me verify this. The distance from the center to the line is ( frac{a}{2} ), and the radius is ( frac{asqrt{3}}{2} ). The length of the chord (the segment inside the sphere) can be calculated using the formula:[text{Length} = 2 sqrt{R^2 - d^2} = 2 sqrt{left( frac{asqrt{3}}{2} right)^2 - left( frac{a}{2} right)^2} = 2 sqrt{frac{3a^2}{4} - frac{a^2}{4}} = 2 sqrt{frac{2a^2}{4}} = 2 cdot frac{asqrt{2}}{2} = asqrt{2}]Yes, that matches the earlier calculation. So, the length of the segment cut off by the sphere from line ( EF ) is ( asqrt{2} ).But wait, in the problem statement, it's mentioned that all vertices are on the sphere, and ( E ) and ( F ) are midpoints. So, the line ( EF ) is inside the cube, and the sphere cuts it at two points outside the segment ( EF ), meaning the chord length is ( asqrt{2} ), which is longer than the segment ( EF ) itself, which is ( a ).But the problem asks for the length of the line segment cut off by the sphere from line ( EF ). If the line ( EF ) is considered as an infinite line, then the segment inside the sphere is ( asqrt{2} ). However, if we consider only the segment between ( E ) and ( F ), which is entirely inside the sphere, then the length is ( a ).But the problem says "the line segment cut off by sphere ( O ) from line ( EF )." The phrase "cut off" suggests that the sphere intersects the line ( EF ) at two points, and the segment between these two points is the one cut off. Therefore, the length should be ( asqrt{2} ).But earlier, when I computed the intersection points, they were at ( t = frac{1 pm sqrt{2}}{2} ), which are outside the segment ( EF ) (since ( t ) ranges from 0 to 1). Therefore, the segment cut off by the sphere is not the segment ( EF ), but a longer segment outside of it.Wait, but in reality, the line ( EF ) is inside the cube, and the sphere is circumscribed around the cube. So, the line ( EF ) is entirely inside the sphere, meaning the sphere doesn't cut off any part of ( EF ); the entire segment ( EF ) is inside the sphere. Therefore, the length cut off by the sphere from line ( EF ) is the entire length of ( EF ), which is ( a ).But this contradicts the earlier calculation where the chord length is ( asqrt{2} ). I think the confusion arises from whether we're considering the segment ( EF ) or the infinite line ( EF ).The problem says "the line segment cut off by sphere ( O ) from line ( EF )." This implies that the sphere intersects the line ( EF ) at two points, and the segment between these two points is the one cut off. Therefore, the length is ( asqrt{2} ).But in our case, the segment ( EF ) is entirely inside the sphere, so the sphere doesn't cut off any part of ( EF ). Therefore, the length cut off is the entire segment ( EF ), which is ( a ).Wait, but according to the formula for the chord length, it's ( asqrt{2} ), which is longer than ( a ). So, perhaps the problem is considering the infinite line ( EF ), and the segment cut off by the sphere is the chord, which is ( asqrt{2} ).But in the cube, the line ( EF ) is entirely inside the sphere, so the chord length should be longer than the segment ( EF ). Therefore, the length cut off by the sphere from line ( EF ) is ( asqrt{2} ).But let me think again. The chord length formula gives the length of the segment of the line inside the sphere. Since the line ( EF ) is entirely inside the sphere, the chord length is the entire line segment ( EF ), which is ( a ). But according to the formula, it's ( asqrt{2} ), which is longer than ( a ). That doesn't make sense because ( asqrt{2} ) is longer than the edge length ( a ), but the chord should be longer than the segment inside the sphere.Wait, no, the chord length is the length of the segment of the line inside the sphere. If the line is entirely inside the sphere, then the chord length is infinite, which is not the case here. Therefore, my earlier approach was incorrect.Wait, no, the chord length formula applies to a line that intersects the sphere at two points, forming a chord. If the line is entirely inside the sphere, then the chord length is the entire line, which is not applicable here because the line is infinite. Therefore, the chord length formula is only applicable when the line intersects the sphere at two points, forming a finite chord.In our case, the line ( EF ) is entirely inside the sphere, so the chord length is not applicable. Therefore, the length cut off by the sphere from line ( EF ) is the entire segment ( EF ), which is ( a ).But this contradicts the earlier calculation where the chord length is ( asqrt{2} ). I think the confusion arises from the interpretation of the problem.Let me re-examine the problem statement: "The length of the line segment cut off by sphere ( O ) from line ( EF ) is __________."The phrase "cut off" implies that the sphere intersects the line ( EF ) at two points, and the segment between these two points is the one cut off. Therefore, the length is the distance between these two intersection points, which is ( asqrt{2} ).But earlier, when I solved for ( t ), the intersection points were at ( t = frac{1 pm sqrt{2}}{2} ), which are outside the segment ( EF ). Therefore, the segment cut off by the sphere is not the segment ( EF ), but a longer segment outside of it.Wait, but in reality, the line ( EF ) is inside the cube, and the sphere is circumscribed around the cube. Therefore, the line ( EF ) is entirely inside the sphere, so the sphere doesn't cut off any part of ( EF ); the entire segment ( EF ) is inside the sphere. Therefore, the length cut off by the sphere from line ( EF ) is the entire length of ( EF ), which is ( a ).But this contradicts the chord length formula, which suggests the length is ( asqrt{2} ). I think the confusion is arising because the line ( EF ) is not a space diagonal, but a face diagonal in a particular plane.Wait, let me think about the cube and the sphere again. The sphere has a radius of ( frac{asqrt{3}}{2} ), and the line ( EF ) is at a distance of ( frac{a}{2} ) from the center. Therefore, the chord length is ( 2 sqrt{R^2 - d^2} = 2 sqrt{left( frac{asqrt{3}}{2} right)^2 - left( frac{a}{2} right)^2} = 2 sqrt{frac{3a^2}{4} - frac{a^2}{4}} = 2 sqrt{frac{2a^2}{4}} = 2 cdot frac{asqrt{2}}{2} = asqrt{2} ).So, the chord length is ( asqrt{2} ), which is the length of the segment of the line ( EF ) that lies inside the sphere. However, since the segment ( EF ) is entirely inside the sphere, the chord length is longer than ( EF ). But that doesn't make sense because ( EF ) is a straight line inside the sphere, so the chord should be the entire line segment ( EF ).Wait, no, the chord length formula gives the length of the segment of the line inside the sphere, which in this case is the entire segment ( EF ) because ( EF ) is entirely inside the sphere. Therefore, the chord length should be equal to the length of ( EF ), which is ( a ). But according to the formula, it's ( asqrt{2} ), which is longer. This inconsistency suggests that there's a mistake in my reasoning.Wait, perhaps I made a mistake in calculating the distance from the center to the line ( EF ). Let me recalculate that.The distance ( d ) from the origin to the line ( EF ) can be calculated using the formula:[d = frac{|vec{OE} times vec{EF}|}{|vec{EF}|}]Where ( vec{OE} ) is the vector from the origin to point ( E ), and ( vec{EF} ) is the direction vector of the line.Given ( E = left( frac{a}{2}, frac{a}{2}, 0 right) ) and ( F = left( -frac{a}{2}, frac{a}{2}, 0 right) ), the vector ( vec{EF} = F - E = left( -a, 0, 0 right) ).The vector ( vec{OE} = left( frac{a}{2}, frac{a}{2}, 0 right) ).The cross product ( vec{OE} times vec{EF} ) is:[vec{OE} times vec{EF} = begin{vmatrix}mathbf{i} & mathbf{j} & mathbf{k} frac{a}{2} & frac{a}{2} & 0 - a & 0 & 0 end{vmatrix}= mathbf{i} left( frac{a}{2} cdot 0 - 0 cdot 0 right) - mathbf{j} left( frac{a}{2} cdot 0 - (-a) cdot 0 right) + mathbf{k} left( frac{a}{2} cdot 0 - (-a) cdot frac{a}{2} right)][= 0mathbf{i} - 0mathbf{j} + left( 0 - (-frac{a^2}{2}) right)mathbf{k} = frac{a^2}{2}mathbf{k}]So, the magnitude of this cross product is ( frac{a^2}{2} ).The magnitude of ( vec{EF} ) is ( sqrt{(-a)^2 + 0^2 + 0^2} = a ).Therefore, the distance ( d ) is:[d = frac{frac{a^2}{2}}{a} = frac{a}{2}]So, the distance from the center to the line ( EF ) is indeed ( frac{a}{2} ), which is less than the radius ( frac{asqrt{3}}{2} ). Therefore, the line ( EF ) intersects the sphere at two points, and the chord length is ( asqrt{2} ).But since the segment ( EF ) is entirely inside the sphere, the chord length ( asqrt{2} ) must be the length of the segment of the line ( EF ) that lies inside the sphere, which is longer than ( EF ) itself. However, this seems contradictory because ( EF ) is a straight line inside the sphere, so the chord length should be the entire segment ( EF ).Wait, no, the chord length is the length of the segment of the line inside the sphere, which in this case is the entire line ( EF ) because ( EF ) is entirely inside the sphere. Therefore, the chord length should be equal to the length of ( EF ), which is ( a ). But according to the formula, it's ( asqrt{2} ), which is longer. This suggests that there's a mistake in the formula application.Wait, no, the chord length formula is correct. The chord length is the length of the segment of the line inside the sphere, which is longer than the segment ( EF ) because ( EF ) is just a part of the line ( EF ). Therefore, the chord length is ( asqrt{2} ), which is the length of the segment of the line ( EF ) that lies inside the sphere, which includes points beyond ( E ) and ( F ).But in the problem statement, it's mentioned that ( E ) and ( F ) are midpoints of edges, so the line ( EF ) is entirely inside the cube, and hence inside the sphere. Therefore, the entire segment ( EF ) is inside the sphere, and the sphere doesn't cut off any part of ( EF ); the entire segment remains inside. Therefore, the length cut off by the sphere from line ( EF ) is the entire length of ( EF ), which is ( a ).But this contradicts the chord length formula, which suggests the length is ( asqrt{2} ). I think the confusion arises from the interpretation of the problem. The problem says "the line segment cut off by sphere ( O ) from line ( EF )." If the line ( EF ) is entirely inside the sphere, then the sphere doesn't cut off any part of ( EF ); the entire segment remains inside. Therefore, the length cut off is zero, which doesn't make sense.Alternatively, if we consider the line ( EF ) as an infinite line, then the sphere cuts it at two points, forming a chord of length ( asqrt{2} ). Therefore, the length cut off by the sphere from line ( EF ) is ( asqrt{2} ).But in the context of the problem, since ( E ) and ( F ) are midpoints of edges of the cube, which is inscribed in the sphere, the line ( EF ) is entirely inside the sphere, so the sphere doesn't cut off any part of ( EF ). Therefore, the length cut off is the entire segment ( EF ), which is ( a ).However, considering the mathematical definition, the chord length is the segment of the line inside the sphere, which in this case is longer than ( EF ). Therefore, the length cut off by the sphere from line ( EF ) is ( asqrt{2} ).But I'm still confused because the segment ( EF ) is entirely inside the sphere, so the sphere doesn't cut off any part of it. Therefore, the length cut off is zero, but that's not the case.Wait, perhaps the problem is considering the line ( EF ) extended beyond ( E ) and ( F ), and the sphere cuts it at two points, forming a chord. Therefore, the length cut off is the chord length ( asqrt{2} ).But in the cube, the line ( EF ) is entirely inside the sphere, so the chord length is the entire line segment ( EF ), which is ( a ). Therefore, the length cut off by the sphere from line ( EF ) is ( a ).But according to the chord length formula, it's ( asqrt{2} ). I think the confusion is arising because the line ( EF ) is not a space diagonal, but a face diagonal, and the chord length is calculated based on the distance from the center to the line.Wait, let me think differently. The line ( EF ) is in the plane ( y = frac{a}{2} ), ( z = 0 ). The intersection of the sphere with this plane is a circle with radius ( sqrt{R^2 - d^2} ), where ( d ) is the distance from the center to the plane.The distance from the center (origin) to the plane ( y = frac{a}{2} ) is ( frac{a}{2} ). Therefore, the radius of the circle is:[sqrt{left( frac{asqrt{3}}{2} right)^2 - left( frac{a}{2} right)^2} = sqrt{frac{3a^2}{4} - frac{a^2}{4}} = sqrt{frac{2a^2}{4}} = frac{asqrt{2}}{2}]Therefore, the intersection of the sphere with the plane ( y = frac{a}{2} ), ( z = 0 ) is a circle with radius ( frac{asqrt{2}}{2} ).The line ( EF ) lies in this plane and is a straight line passing through the circle. The length of the chord (the segment of the line inside the sphere) is:[2 times frac{asqrt{2}}{2} = asqrt{2}]Therefore, the length of the line segment cut off by the sphere from line ( EF ) is ( asqrt{2} ).But in the cube, the line ( EF ) is entirely inside the sphere, so the chord length is the entire segment ( EF ), which is ( a ). Therefore, the length cut off by the sphere is ( a ).Wait, but according to the circle intersection, the chord length is ( asqrt{2} ), which is longer than ( a ). This suggests that the line ( EF ) extends beyond the cube, and the sphere cuts it at two points, forming a chord of length ( asqrt{2} ). Therefore, the length cut off by the sphere from line ( EF ) is ( asqrt{2} ).But in the problem statement, it's mentioned that all vertices are on the sphere, and ( E ) and ( F ) are midpoints of edges. Therefore, the line ( EF ) is entirely inside the cube and hence inside the sphere. Therefore, the sphere doesn't cut off any part of ( EF ); the entire segment remains inside. Therefore, the length cut off is the entire segment ( EF ), which is ( a ).But this contradicts the mathematical calculation. I think the confusion arises from the interpretation of the problem. The problem says "the line segment cut off by sphere ( O ) from line ( EF )." If the line ( EF ) is entirely inside the sphere, then the sphere doesn't cut off any part of ( EF ); the entire segment remains inside. Therefore, the length cut off is zero, which doesn't make sense.Alternatively, if we consider the line ( EF ) as an infinite line, then the sphere cuts it at two points, forming a chord of length ( asqrt{2} ). Therefore, the length cut off by the sphere from line ( EF ) is ( asqrt{2} ).But in the context of the cube, the line ( EF ) is entirely inside the sphere, so the sphere doesn't cut off any part of ( EF ); the entire segment remains inside. Therefore, the length cut off is the entire segment ( EF ), which is ( a ).However, according to the mathematical calculation, the chord length is ( asqrt{2} ), which is longer than ( a ). Therefore, the correct answer should be ( asqrt{2} ).But I'm still unsure because the segment ( EF ) is entirely inside the sphere, so the sphere doesn't cut off any part of it. Therefore, the length cut off is zero, but that's not the case.Wait, perhaps the problem is considering the line ( EF ) extended beyond the cube, and the sphere cuts it at two points, forming a chord. Therefore, the length cut off by the sphere from line ( EF ) is ( asqrt{2} ).But in the problem statement, it's mentioned that all vertices are on the sphere, and ( E ) and ( F ) are midpoints of edges. Therefore, the line ( EF ) is entirely inside the sphere, so the sphere doesn't cut off any part of ( EF ); the entire segment remains inside. Therefore, the length cut off is the entire segment ( EF ), which is ( a ).But according to the chord length formula, it's ( asqrt{2} ). I think the confusion arises from the interpretation of the problem. The problem says "the line segment cut off by sphere ( O ) from line ( EF )." If the line ( EF ) is entirely inside the sphere, then the sphere doesn't cut off any part of ( EF ); the entire segment remains inside. Therefore, the length cut off is zero, which doesn't make sense.Alternatively, if we consider the line ( EF ) as an infinite line, then the sphere cuts it at two points, forming a chord of length ( asqrt{2} ). Therefore, the length cut off by the sphere from line ( EF ) is ( asqrt{2} ).But in the context of the cube, the line ( EF ) is entirely inside the sphere, so the sphere doesn't cut off any part of ( EF ); the entire segment remains inside. Therefore, the length cut off is the entire segment ( EF ), which is ( a ).However, the mathematical calculation suggests the chord length is ( asqrt{2} ). I think the correct answer is ( asqrt{2} ), considering the line ( EF ) as an infinite line intersecting the sphere at two points, forming a chord of length ( asqrt{2} ).Therefore, the length of the line segment cut off by sphere ( O ) from line ( EF ) is ( asqrt{2} ).</think>